Infectious Disease ER CAQ

Lakukan tugas rumah & ujian kamu dengan baik sekarang menggunakan Quizwiz!

Question: How long does Bacillus cereus food poisoning typically last?

Answer: 24-48 hours

Question: What percentage of patients with a tick-borne illness recall a tick bite?

Answer: 50-70%.

Question: What is the most common viral cause of rhabdomyolysis?

Answer: Influenza.

Question: What is the recommended treatment for Tularemia?

Answer: Streptomycin.

In an adult patient who is otherwise asymptomatic and identifies an engorged tick that has been attached for longer than 36 hours, what is the most appropriate medication for prophylaxis of Lyme disease? Clindamycin Doxycycline Penicillin Vancomycin

Correct Answer ( B ) Explanation: Lyme disease is transmitted by the bite of an Ixodes scapularis tick and is the most common vectorborne disease in the United States. Doxycycline is the drug of choice for prophylaxis against Lyme disease in an adult patient who is not pregnant and living in an endemic area. Nadelman and colleagues have reported that the administration of the medication within 72 hours of the time the tick was removed significantly reduces the likelihood of contracting Lyme disease.

10-day-old boy presents to the ED with copious purulent drainage from his right eye. He was the product of a normal spontaneous vaginal delivery. One day after delivery, Mom developed vaginal irritation. Which of the following is the most likely etiology of the patient's presentation? Chemical Chlamydia trachomatis Herpes simplex virus N. gonorrhoeae

Correct Answer ( B ) Explanation: There is strong clinical evidence that this patient's conjunctivitis is infectious in etiology. The timing of presentation following birth and maternal findings are useful in determining the etiology. The "Rule of 5s" helps to predict the most likely bacterial cause. This patient's symptoms began 10 days after delivery, making the most likely etiology C. trachomatis.

Which of the following is true regarding botulism? Foodborne botulism develops 10-14 days following toxin ingestion Foodborne botulism is caused by ingestion of a heat-stable toxin (type A) Infantile botulism is the most common form Intestinal colonization is common in foodborne botulism

Correct Answer ( C ) Explanation: First described in the mid-1970s, infantile botulism is now the most common form of botulism in the United States. Children typically present with poor feeding, decreased suckling, loss of facial expression, constipation, and noticeable neck and peripheral weakness—a constellation of symptoms known as "floppy baby syndrome." It occurs in children younger than 12 months of age, with a peak incidence at 3 months. It has been associated with ingestion of honey, corn syrup, and vacuum or environmental dust. It is caused by the ingestion and subsequent intestinal colonization of Clostridium botulinum spores in the intestinal tract.

Which of the following is true regarding tetanus prophylaxis? Patients older than 7 years of age should receive diphtheria-tetanus-acellular pertussis (DTaP or DT) prophylaxis Patients younger than 7 years of age should receive tetanus-diphtheria (Td) or tetanus-diphtheria-acellular pertussis (Tdap) prophylaxis Tetanus immune globulin (TIG) prophylaxis is recommended for unimmunized patients with a high-risk wound Tetanus immune globulin (TIG) prophylaxis should be given at the same site as tetanus-diphtheria (Td) or tetanus-diphtheria-acellular pertussis (Tdap) prophylaxis

Correct Answer ( C ) Explanation: There are several different tetanus vaccine preparations, often given in combination with diphtheria (D or d component) and pertussis (the ap or aP component). These include DTP, DTap, Td, and Tdap. DTaP is the preferred vaccine for primary pediatric immunization. Uppercase indicates a larger concentration of the component, whereas lowercase indicates a smaller concentration. Tetanus vaccination is recommended for all ED patients with wounds. However, higher-risk tetanus-prone wounds include those more than six hours old, greater than 1 cm in depth, contaminated with dirt or soil, stellate, denervated, ischemic, or obviously infected. Adults with an uncertain childhood history of vaccination should receive a complete primary immunization series. In the presence of a high-risk wound, these patients should also receive a dose of human tetanus immunoglobulin (TIG) at a separate injection site. Previously immunized healthy adults should receive a Td or Tdap booster if it has been 10 or more years since their last dose. Immunity may wane more quickly in certain high-risk patients such as IV drug abusers, patients with immunosuppression, and the elderly, thus necessitating more frequent booster doses. The American Congress of Obstetricians and Gynecologists updated their guidelines and recommends Tdap during each pregnancy, irrespective of the patient's prior history of receiving Tdap. The new guidance was issued based on an imperative to minimize the significant burden of pertussis disease in vulnerable newborns, the reassuring safety data on the use of Tdap in adults, and the evolving immunogenicity data that demonstrate considerable waning of immunity after immunization.

patients with sickle cell disease, exposure to which of the following viruses can result in an aplastic crisis? Coxsackie A 16 Human Herpes Virus 6 Paramyxovirus Parvovirus B19

Correct Answer ( D ) Explanation: Aplastic crisis is defined as having a reticulocyte count of less than 1%. Patients with sickle cell disease who are infected with Parvovirus B19 are at risk for developing an aplastic crisis. Parvovirus B19 is the causative agent of Fifth disease and can lead to an aplastic crisis by causing a temporary arrest of red blood cell production. This is characterized by a sudden decrease in hemoglobin production by bone marrow resulting in severe anemia. Patients may present with pallor, lethargy, and shock. Treatment includes hemodynamic support and blood transfusion.

Question: What is "Blackwater" fever?

Answer: "Blackwater" fever describes the presence of hemoglobinuria resulting from hemolytic anemia in malaria.

Question: HIV postexposure prophylaxis should ideally be initiated within what time frame?

Answer: 1 to 2 hours.

Question: What is the choroidal tubercle?

Answer: A granuloma in the choroid of the retina that is specific for disseminated TB.

Question: What is the most common neurological symptom from second-stage Lyme disease?

Answer: A unilateral or bilateral facial nerve palsy.

Question: What is the leading cause of mortality in sickle cell patients?

Answer: Acute Chest Syndrome (ACS).

Question: Where should immune globulin be given in postexposure prophylaxis?

Answer: Adjacent to the wound site. However, for patients with no visualized wound (such as those with a bat exposure but no bite), any intramuscular location is acceptable.

Question: What is the treatment of a pregnant patient with syphilis and a penicillin allergy?

Answer: Admission for penicillin desensitization as only penicillin is effective in these patients.

Question: What is the first-line therapy for patients diagnosed with cryptococcal meningitis?

Answer: Amphotericin B.

Question: What antibiotics are associated with the development of a morbilliform rash in a patient with infectious mononucleosis?

Answer: Ampicillin and amoxicillin.

Question: What is the empiric treatment of choice of gram-negative bacteremia in an immunocompetent patient with a history of health care exposure (e.g. a nursing home patient)?

Answer: An antipseudomonal cephalosporin, such as cefepime or ceftazidime.

Question: What is the most common clinical presentation of late Lyme disease?

Answer: Arthritis.

Question: What is the most common complication of Rubella infection?

Answer: Arthropathies and arthritis. Rubella (German Measles) Patient will be complaining of fever, sore throat, headache PE will show prominent lymphadenopathy (occipital, cervical, postauricular) and a rash that spread from face down Comments: 1st trimester pregnancy-congenital defects In 2004, rubella was officially declared eliminated from the United States and from the Americas in 2015.

Question: What is the typical distribution of weakness caused by West Nile meningoencephalitis?

Answer: Asymmetric; isolated facial involvement may also be seen.

Question: What organism causes food poisoning after the ingestion of fried rice?

Answer: Bacillus cereus.

Question: What is the treatment for primary syphilis?

Answer: Benzathine penicillin G 2.4 million units IM single dose is nearly universally curative, while doxycycline for two weeks is an alternative for penicillin-allergic patients. Syphilis Primary: painless chancre Secondary: lymphadenopathy, condyloma lata, rash on palms/soles Tertiary: gummas VDRL and RPR positive 1-4 weeks after infection Primary/secondary: IM benzathine penicillin G x 1 dose Tertiary: IM benzathine penicillin G qwk x 3 weeks

Question: What is the deep tendon response in patients with botulinum poisoning?

Answer: Botulinum toxin prevents the release of acetylcholine from motor neurons resulting in flaccid paralysis and diminished deep tendon reflexes.

Question: What is the most common infectous cause of blindness worldwide?

Answer: C. trachomatis conjunctivitis. Chlamydial Conjunctivitis Patient will be a neonate 5-14 days after delivery PE will show mucopurulent ocular discharge, eyelid swelling, and erythematous conjunctiva Diagnosis is made by culture Most commonly caused by Chlamydia trachomatis Treatment is oral erythromycin

Question: An ophthalmoscopic finding of fluffy white perivascular lesions in an AIDS patient with a CD4 count less than 50 cells/μL suggests what diagnosis?

Answer: CMV retinitis.

Question: What intravenous antibiotics are used for severe, disseminated or late Lyme disease?

Answer: Ceftriaxone or penicillin.

Question: What other antibiotic can be used to treat Lyme disease in a pregnant patient?

Answer: Cefuroxime axetil, a second-generation oral cephalopsporin.

Question: Which disease is transmitted by the reduviid bug?

Answer: Chagas disease.

Question: What tick-borne illness is characterized by a biphasic fever pattern?

Answer: Colorado tick fever.

Question: What common CNS infection in patients with Acquired Immunodeficiency Syndrome (AIDS) is treated with Amphotericin B?

Answer: Cryptococcal meningitis.

Question: What is the appearance of Cryptococcus neoformans on India ink preparation?

Answer: Cryptococcus appears as round encapsulated yeast.

Question: What late skin finding is commonly associated with staphylococcal toxic shock syndrome?

Answer: Desquamation of palms and soles one to two weeks after onset of symptoms. Toxic Shock Syndrome S. aureus Tampons, nasal packs, wounds Hypotension, high fever, diffuse erythroderma, multisystem organ dysfunction Identify/remove source

Question: What is the Rumpel-Leede test?

Answer: Development of petechiae after placing a tourniquet on the arm of a patient with Rocky Mountain spotted fever.

Question: What is the dose and duration of treatment of doxycycline for Lyme disease prevention?

Answer: Doxycycline 200 mg orally once. Lyme Disease Patient with a history of being in the woods hiking or camping Complaining of:Stage I: erythema migrans (pathognomonic), viral-like syndrome (fever, fatigue, malaise, myalgia, headache)Stage II: myocarditis, bilateral Bell palsyStage III: chronic arthritis, chronic encephalopathy PE will show slightly raised red lesion with central clearing, erythema migrans (bull's-eye) rash Most commonly caused by Borrelia burgdorferi carried by Ixodes tick Treatment is doxycycline, children - amoxicillin or doxycycline (if used for < 21 days), pregnant - amoxicillin Comments: Bilateral facial nerve palsy is virtually pathognomonic for Lyme disease

Question: What bacteria is associated with outbreaks of ground beef related hemorrhagic colitis?

Answer: E. coli O157:H7

Question: What is the name of the rash associated with Lyme disease?

Answer: Erythema migrans—described as a circular lesion with central clearing or erythema at the site of a tick bite.

Question: What factors can result in an erythrocyte sedimentation rate that is lower than anticipated?

Answer: Erythrocyte abnormalities such as sickle cell disease, extreme leukocytosis, and heart failure.

Question: True or false: Piperacillin-tazobactam provides poor coverage for anaerobic bacteria.

Answer: False. Necrotizing Fasciitis Type 1: polymicrobial Type 2: Group A streptococcal infection Pain out of proportion to exam Systemic signs of infection Cellulitis turns dusky blue with bullae/vesicles Radiograph: subcutaenous emphyesma Emergent surgical debridement

Question: True or false: A patient diagnosed with influenza B will gain the same benefits from antiviral therapy as patients with influenza A.

Answer: False. Studies have not shown the same benefit in using antiviral therapy for patients with influenza B.

Question: Which class of antibiotics is no longer recommended for treatment of gonorrhea due to bacterial resistance?

Answer: Fluoroquinolones.

Question: How long should patients diagnosed with chlamydia or gonorrhea be counseled to avoid sexual contact for?

Answer: For 7 days after completion of antibiotic treatment.

Question: What are the four types of clinical tetanus?

Answer: Generalized, localized, cephalic and neonatal.

Question: Should you order serologic testing to diagnose toxoplasmosis?

Answer: Generally, no, because antibodies are fairly prevalent in the general population.

Question: What is the most common cause of septic arthritis in patients under 50 years of age?

Answer: Gonococcus arthritis.

Question: How is the Measles virus spread?

Answer: It is spread through infectious droplets (that can remain airborne for up to 2 hours) and is highly contagious.

Question: How long after symptoms cease do patients with genital herpes stop shedding virus?

Answer: It is suspected that patients with a history of herpes can shed virus continuously, whether symptoms are present or not.

Question: What virus causes progressive multifocal leukoencephalopathy, a demyelinating disease in patients with HIV?

Answer: JC virus.

Question: What laboratory test is commonly significantly elevated in patients with cyanide toxicity?

Answer: Lactic acid.

Question: What mycobacterial infection is suggested by skin lesions and neuropathy

Answer: Leprosy.

Question: What is the most commonly identified source of infection in septic patients?

Answer: Lungs (respiratory system).

Question: What are the first-line antimicrobials for amebiasis with a liver abscess?

Answer: Metronidazole for 10 days followed by iodoquinol for 20 days. Food Poisoning Bacillus cereus - eating reheated rice - diarrhea C. botulinum - canned food - diarrhea Clostridium perfringens - reheated meat or canned foods within 24 hours - watery diarrhea and epigastric pain Ciguatera - sea bass, grouper, red snapper - diarrhea E. coli O157:H7 - undercooked meat - diarrhea, HUS Salmonella: poultry, meat, eggs Scombroid - peppery tasting fish - diarrhea Staphylococcus aureus - eating meats, mayonnaise, custard 1 - 6 hours ago - nausea, vomiting, abdominal pain and diarrhea V. parahaemolyticus, V. vulnificus - shellfish and seafood - diarrhea Yersinia: undercooked pork - pseudoappendicitis, bloody diarrhea

Question: What finding on a Tzank smear confirms the diagnosis of an HSV infection?

Answer: Multinucleated giant cells.

Question: What is the most common complication of meningococcemia?

Answer: Myocarditis with congestive heart failure or conduction abnormalities.

Question: What are 3 rare complications of hand-foot-and-mouth disease?

Answer: Myocarditis, pneumonia, meningoencephalitis.

Question: Is pregnancy a contraindication to receiving either the rabies immunoglobulin or the rabies vaccine?

Answer: No.

Question: Does the Jarisch-Herxheimer reaction occur during the treatment of tertiary syphilis?

Answer: No. It is limited to primary and secondary syphilis.

Question: Do patients with lymphocutaneous sporotrichosis require admission?

Answer: No; unless there is disseminated disease or involvement of joints, bones, or tendons, it can be treated as an outpatient.

Question: What classic diffuse spasm pattern is associated with tetanus and hallmarked by arching of the back?

Answer: Opisthotonus.

One Step Further Question: How is Rocky Mountain spotted fever diagnosed?

Answer: PCR can be used initially, and serial serologic examinations by indirect fluorescent antibody confirm the diagnosis, retrospectively.

Question: Why has metronidazole replaced penicillin as the antibiotic of choice to treat tetanus?

Answer: Pencilin has GABA-antagonistic activity and may potentiate the effects of tetanospasmin.

Question: During which stage of syphilis may testing be negative?

Answer: Primary as it may take between 1 to 4 weeks for tests to become positive.

Question: When do Koplik's spots occur in relation to the rash of measles?

Answer: Prior to the onset of rash.

Question: Which form of anthrax (cutaneous, gastrointestinal, pulmonary) is the most lethal?

Answer: Pulmonary anthrax is the most lethal form of anthrax.

Question: What syndrome causes a vesicular rash in or around the ear canal, facial paralysis and hearing loss?

Answer: Ramsay hunt syndrome caused by zoster.

Question: What is the definition of septic shock?

Answer: Sepsis with hypotension not responsive to fluids.

Question: What are the best tests for confirmation of Salmonella enteritis?

Answer: Stool cultures and PCR assay.

Question: Infection with what organism is associated with atypical (non-diarrheal) hemolytic uremic syndrome?

Answer: Streptococcus pneumoniae.

Question: What is the treatment for botulism?

Answer: Supportive care and equine botulinum antitoxin.

Question: What is the vector that spreads malaria?

Answer: The female Anopheles mosquito.

Question: Which muscle becomes unusually tender in patients with Rocky Mountain spotted fever?

Answer: The gastrocnemius muscle.

Question: Which part of the spinal cord is generally affected in tertiary syphilis and leads to gait instability, called a tabetic gait?

Answer: The neural tracts primarily affected by tertiary syphilis include the posterior columns (dorsal columns) of the spinal cord.

Question: What is a Jarisch-Herxheimer reaction?

Answer: The onset of fever, myalgias, headache, tachycardia, and tachypnea after initiation of antibacterial treatment of a spirochete illness

Question: What is the definition of Sepsis Criteria?

Answer: The presence of SIRS + a suspected or confirmed source of infection. Sepsis Syndromes SIRS: 2 or more of the followingT >38° or <36°HR > 90 bpmRR > 20 bpm or PCO2 < 32 mmHgWBC > 12,000 or < 4000 Sepsis: SIRS + infection Severe sepsis: sepsis + organ dysfunction Septic shock: sepsis + refractory hypotension

Question: Seizures that occur in those diagnosed with herpes simplex meningoencephalitis are thought to originate from what part of the brain?

Answer: The temporal lobe.

Question: What laboratory abnormalities are characteristic of Rocky Mountain spotted fever?

Answer: Thrombocytopenia and hyponatremia.

Question: What is the most common abnormality seen on a complete blood count in hantavirus infection?

Answer: Thrombocytopenia.

Question: What is a known complication of infection with parvovirus B19 in children with sickle cell disease?

Answer: Transient aplastic crisis.

Question: True or false - Blood cultures that show growth starting after 72 hours of incubation are more likely to represent contamination?

Answer: True.

Question: How long should a deer tick be attached before prophylactic treatment to prevent Lyme disease is recommended?

Answer: Typically, longer than 36 hours. Lyme Disease Patient with a history of being in the woods hiking or camping Complaining of:Stage I: erythema migrans (pathognomonic), viral-like syndrome (fever, fatigue, malaise, myalgia, headache)Stage II: myocarditis, bilateral Bell palsyStage III: chronic arthritis, chronic encephalopathy PE will show slightly raised red lesion with central clearing, erythema migrans (bull's-eye) rash Most commonly caused by Borrelia burgdorferi carried by Ixodes tick Treatment is doxycycline, children - amoxicillin or doxycycline (if used for < 21 days), pregnant - amoxicillin Comments: Bilateral facial nerve palsy is virtually pathognomonic for Lyme disease

Question: What findings are typically seen on CSF analysis in tuberculous meningitis?

Answer: Very low glucose, elevated protein and a slightly elevated opening pressure.

Which of the following foodborne bacteria causes illness through a preformed toxin? Bacillus cereus Escherichia coli Salmonella typhi Vibrio vulnificus

Correct Answer ( A ) Explanation: Bacillus cereus is a foodborne spore-forming bacterium that causes illness due to preformed toxins. These toxins are present in the food prior to ingestion and stimulate the host immune system to release inflammatory cytokines, resulting in nausea, vomiting, and mild nonbloody diarrhea. Because the toxins responsible for the illness are present prior to ingestion, onset of symptoms is rapid (1 to 6 hours). Other foodborne bacteria that also cause illness through a preformed toxin include Staphylococcus aureus and Clostridium perfringens. B. cereus food poisoning is typically self-limited and treatment is supportive. Oral rehydration, if tolerated, is preferred to parenteral fluids.

What is the most likely cause of a patient presenting with bilateral facial nerve palsy? Borrelia burgdorferi Herpes simplex virus Leukemia Mycoplasma pneumoniae

Correct Answer ( A ) Explanation: Bell's palsy is a paralysis of the peripheral seventh nerve. Clinically, one is able to distinguish a peripheral from central cause of facial motor weakness by involvement of the forehead in a peripheral palsy. The forehead receives bilateral innervation centrally and a stroke syndrome will not interfere with upper facial strength because of the contralateral innervation. In Bell's palsy, the seventh nerve is affected after leaving the brainstem and weakness is seen on the entire face. Bilateral Bell's palsy occurs simultaneously very rarely. Borrelia burgdorferi is the causative organism of Lyme disease and Lyme is one of the few causes of bilateral Bell's palsy. Other causes include Guillan-barre syndrome, sarcoidosis, meningitis, or bilateral neurofibromas.

A 26-year-old man presents with a severe retro-orbital headache, a sudden-onset fever of 103.3°F, nausea, and severe myalgias six days after returning from Panama. On exam, he has a morbiliform rash on his abdomen and back. Which of the following is most likely responsible for his symptoms? Dengue fever Japanese encephalitis Malaria Yellow fever

Correct Answer ( A ) Explanation: Dengue fever is the second most important tropical, febrile illness after malaria. Dengue has a short incubation period of four to seven days. It is transmitted by the Aedes aegypti mosquito. Travelers returning from Africa, the Americas and the Indian subcontinent tend to have classic dengue. Those returning from Southeast Asia usually have the hemorrhagic variant. Classic dengue manifests as sudden onset of high fever, with retro-orbital headache, nausea, vomiting, severe myalgias, and a rash. It is also known as "breakbone fever" due to the severe myalgias. Hemorrhagic dengue can result in death due to fever and shock. Diagnosis is clinical and confirmed with ELISA for IgM. Treatment is supportive with both variants.

45-year-old man with a known history of HIV presents to the emergency department with fever, weight loss, night sweats and diarrhea. He states he has been off his antiretroviral therapy for the last 6 months. On physical exam he appears cachectic. He is febrile with diffuse, tender lymphadenopathy and hepatosplenomegaly. Laboratory analysis reveals a CD4 count less than 50 cells/μL. What is the most likely diagnosis? Disseminated Mycobacterium avium complex Histoplasmosis Pneumocystis jirovecii pneumonia Toxoplasmosis

Correct Answer ( A ) Explanation: Disseminated Mycobacterium avium complex is an opportunistic infection that occurs in patients with acquired immune deficiency syndrome (AIDS) who have a CD4 count less than 50 cells/μL. Patients will often present with nonspecific symptoms such as fever, fatigue, weight loss, night sweats, anorexia, abdominal pain or diarrhea. It can often be mistaken for progression of other HIV-related illnesses. Hepatosplenomegaly and lymphadenopathy may be present on exam. Laboratory analysis most often reveals a significant anemia as well as leukopenia, elevated alkaline phosphatase and decreased albumin. The diagnosis is made with a positive blood culture. Preferred treatment is with clarithromycin and ethambutol. Some clinicians will also add rifabutin. HIV-infected adults and adolescents should receive prophylaxis against disseminated Mycobacterium avium complex disease if they have CD4 counts < 50 cells/μL. Prophylaxis is with clarithromycin or azithromycin.

51-year-old diabetic man presents with perineal pain and fever. Physical examination reveals crepitus along the scrotum and left inner thigh. Which of the following is the best next step in management? Clindamycin, vancomycin, ciprofloxacin plus surgical consultation Hyperbaric oxygen therapy Immediate operative debridement followed by antibiotics Urologic consultation plus broad-spectrum oral antibiotics

Correct Answer ( A ) Explanation: Gas gangrene requires prompt diagnosis and early aggressive management to improve mortality and morbidity. Clindamycin, vancomycin, ciprofloxacin plus surgical consultation are appropriate and sufficient for patients with gas gangrene. It can be caused by Clostridium organisms (i.e. clostridial myonecrosis) or from polymicrobial deep soft tissue infection from wounds or other damage to soft tissue. Clostridium perfringens is the most common organism implicated in gas gangrene. Diabetic patients or patients with immune compromise who present with a perineal necrotizing soft tissue infection have Fournier's gangrene. Patients with gas gangrene typically appear toxic and may report pain out of proportion to examination findings. Crepitus may also be palpated in the affected area, which indicates the presence of subcutaneous emphysema. Plain radiography may reveal subcutaneous gas produced by gas-forming bacteria. In addition to parenteral antibiotics and early surgical debridement, hyperbaric oxygen therapy may be considered for necrotizing soft tissue infections as it causes suppression of growth of anaerobic organisms. Necrotizing Fasciitis Type 1: polymicrobial Type 2: Group A streptococcal infection Pain out of proportion to exam Systemic signs of infection Cellulitis turns dusky blue with bullae/vesicles Radiograph: subcutaenous emphyesma Emergent surgical debridement

A nursing home patient was directed to the emergency department for further evaluation after a blood culture result revealed gram negative bacilli. Infection with which of the following organisms is most likely? AEscherichia coli BHaemophilus influenzae CListeria monocytogenes DNeisseria meningitidis

Correct Answer ( A ) Explanation: Gram-negative bacteremia is a significant problem for patients in both the hospital and community setting. Treatment of gram-negative bacteremia depends on timely recognition and appropriate antibiotic coverage as the mortality rate of gram-negative bacillary sepsis can be as high as 40%. Among hospitalized and critically ill patients, the most common source of gram-negative bacteremia includes the respiratory tract and central venous catheters. Patients in the community setting and in nursing homes are more likely to have the urinary tract identified as the source of infection. Of the possible organisms involved, Escherichia coli is the most commonly implicated bacteria, especially in patients from the community or nursing home settings. Other possible pathogens include P. aeruginosa, K. pneumoniae, and other Enterobacter species.

An 8-year old girl is brought to the emergency department by her mother for abdominal pain, vomiting, and diarrhea for five days. The child's grandmother noticed gross blood in the patient's stool this morning. The patient's vital signs on arrival are 38.4℃, HR 120, BP 102/60, RR 20. Physical examination reveals dry mucous membranes, diffuse abdominal tenderness to palpation without rigidity or rebound, and a non-blanching macular rash on the child's lower extremities. Which of the following is the most likely diagnosis in this patient? Hemolytic uremic syndrome Henoch-Schonlein purpura Rocky Mountain spotted fever Thrombotic thrombocytopenic purpura

Correct Answer ( A ) Explanation: Hemolytic uremic syndrome (HUS) is a multi-system disorder that results in acute renal insufficiency, consumptive thrombocytopenia, and microangiopathic hemolytic anemia. Typical HUS is caused by infection with a Shiga toxin-producing strain of Escherichia coli (such as E. coli O157:H7). This is the predominant etiology in children, causing approximately 90% of cases. Infection is caused by consumption of undercooked beef, unpasteurized milk, or raw fruits and vegetables. The toxin responsible for HUS is produced after ingestion of the bacteria and leads to arrest of protein production in the cells of the glomeruli. This creates a thrombogenic environment that leads to consumptive thrombocytopenia, hemolytic anemia, and renal insufficiency. Diarrheal HUS typically presents with abdominal pain, nausea, vomiting, and inflammatory (bloody) diarrhea. Thrombocytopenia with a petechial rash, anemia, oliguria, seizures and encephalopathy may develop. The degree of renal involvement in HUS is variable and ranges from mild renal insufficiency to renal failure requiring dialysis. An elevated creatinine and azotemia are seen. Approximately 50% of pediatric patients with HUS will require dialysis and dehydration increases the risk and duration of renal support.

What is the most common cause of viral pneumonia in adults? Influenza virus Metapneumovirus Parainfluenza virus Respiratory syncytial virus

Correct Answer ( A ) Explanation: Influenza is the most common viral cause of pneumonia in adults. Influenza is often a self-limited viral illness that generally occurs in outbreaks during the winter months. Influenza most commonly manifests as fever, cough, sore throat, malaise, fatigue, myalgias, headache, conjunctivitis and coryza. However, influenza can also progress to a viral pneumonia. Pneumonia is the most common serious complication of influenza infection in the adult population, particularly in the elderly and in those with chronic medical conditions. This includes both a primary viral pneumonia and secondary bacterial pneumonia.

Which of the following statements is true regarding the diagnosis of Epstein-Barr virus infection? Guillain-Barré syndrome is a possible complication Neutrophilia predominates Splenomegaly occurs in 10% of patients The virus is transmitted via respiratory droplets

Correct Answer ( A ) Explanation: The Epstein-Barr virus (EBV) is implicated in a variety of human illnesses. It is associated with infectious mononucleosis, B-cell lymphoma, Hodgkin disease, Burkitt lymphoma, and nasopharyngeal carcinoma. EBV can affect nearly all organ systems. Neurologic complications such as encephalitis, meningitis, and Guillain-Barré have been reported. EBV is associated with lymphocytosis (B) with > 50% lymphocytes. Atypical lymphocytes are found on examination of the peripheral blood smear. Splenomegaly (C) occurs in > 50% of patients. Therefore, patients should be advised to avoid all contact sports for a minimum of four weeks after illness onset to avoid splenic injury. EBV is transmitted via salivary secretions (D) and requires close contact for transmission (hence lay application of the term "kissing disease"). The infection is usually contracted from an asymptomatic individual who sheds the virus. After infecting the oropharyngeal epithelium, it disseminates through the blood stream. The virus infects B lymphocytes and causes an increase in T lymphocytes, which results in enlargement of lymphoid tissue. In immunocompromised patients with decreased T-cell function, B cells continue to proliferate, and proliferation may lead to neoplastic transformation.

32-year-old previously healthy woman presents to the Emergency Department with fever, headache, lethargy and altered mental status. Her family member states her symptoms began yesterday. She has no known drug allergies. Physical exam reveals a temperature of 102.5°F, nuchal rigidity and she is oriented to herself only. A CT scan of the head is completed and it is normal. A lumbar puncture is performed. Dexamethasone, ceftriaxone and vancomycin are administered. Cerebrospinal fluid studies reveal 450/μL white blood cells with a differential of 96% lymphocytes, glucose 72 mg/dL, protein 80 mg/dL and erythrocytes 450/μL. Which of the following is most appropriate to administer next? Acyclovir Amphotericin B Ampicillin Chloramphenicol

Correct Answer ( A ) Explanation: The cerebrospinal fluid results reveal a pleocytosis with lymphocytic predominance along with elevated protein and a normal glucose, which is indicative of a viral etiology. The elevated erythrocyte level more specifically suggests the diagnosis of herpes simplex meningoencephalitis. The most common symptoms of herpes simplex meningoencephalitis are fever, altered mental status, decreased level of consciousness and focal neurologic findings. HSV encephalitis is a devastating infection of the central nervous system. Even with early administration of therapy after onset of disease, nearly two-thirds of survivors will have significant neurologic deficits. For this reason, empiric therapy with IV acyclovir (10 mg/kg IV every 8 hours) should be initiated as soon as the diagnosis is considered.

Which of the following is true regarding the condition seen in the image of the oral cavity above? The causative organism is spread via the fecal-oral route The illness occurs most frequently in the winter months Treatment is with antiviral medications Vaccination prevents the disease

Correct Answer ( A ) Explanation: The lesions are manifestations of hand-foot-and-mouth disease, a viral infection caused by coxsackievirus. Toddlers and school-age children are most commonly affected. Transmission is by the fecal-oral route and usually occurs in the summer and fall months in crowded places where children congregate such as a swimming pool. It is characterized by a prodrome of fever, malaise, sore throat, and anorexia over a couple of days, followed by the appearance of the characteristic rash. The location of the lesions involve the following: (1) hands and palms (dorsal and palmar surface, sides of fingers); (2) sides of feet and toes, soles (plantar surface); (3) usually the anterior portion of the mouth, most frequently the tongue and buccal mucosa, hard palate, gingivae, and lips. The most frequent site of the lesions is the mouth and the hands are more frequently involved than the feet. The mouth lesions begin as small red macules or papules that turn into vesicles then ulcerate and crust. The extremity lesions are vesicular and pink to red in appearance.They are typically distributed bilaterally and symmetrically on the hands and feet. The skin lesions may be asymptomatic or painful, while the mouth lesions are almost always painful. Treatment is supportive with oral fluids and antipyretics. In most cases, the course is self-limited, resolving in 7-10 days. Herpangina is a characteristic enanthem produced by several enteroviruses, with coxsackie A, B, and echovirus being the most common. This condition is characterized by oral lesions that usually appear in the posterior aspect of the mouth, specifically on the anterior tonsillar pillars (most common), posterior pharyngeal wall, soft palate, tonsils, uvula, and occasionally the posterior buccal mucosa. Herpangitis is rarely associated with aseptic meningitis or other severe enteroviral illnesses.

25-year-old man with a past medical history of acquired immunodeficiency syndrome presents with fever, confusion and neck stiffness. Kernig and Brudzinski tests are positive. A lumbar puncture is performed with an opening pressure > 55 cm H2O. Cerebrospinal fluid studies grow budding yeast with large capsules under India ink stain. Which of the following is the recommended therapy? Amphotericin B and flucytosine Ampicillin and cefotaxime Ceftriaxone and dexamethasone Dexamethasone and flucytosine

Correct Answer ( A ) Explanation: The most common cause of fungal meningitis is Cryptococcus neoformans. The second most common cause is Coccidioides immitis. Fungal meningitis is more likely to occur in patients with acquired immunodeficiency syndrome with a CD4+ T cell count less than 100 cells/mm3 or those taking immunosuppressant therapy. It is the leading cause of mortality among patients with HIV. Diagnosis occurs through cerebrospinal fluid analysis which reveals an elevated opening pressure, a lymphocyte predominance, negative gram stain, low glucose and elevated protein. Cryptococcal meningitis characteristically has markedly elevated opening pressures. A serum cryptococcal antigen test may be sent, and India ink stain may be used to visualize the prominent capsule seen around the organism on microscopy. Without treatment, cryptococcal meningitis is fatal. Cryptococcal meningitis is treated with amphotericin B and flucytosine. In addition to early aggressive antifungal therapy, serial lumbar punctures are often needed for elevated intracranial pressure.

47-year-old man with a history of HIV whose last CD4 count was 95 presents with one week of headache, fever and malaise. He reports associated nausea and vomiting. His temperature is 101°F. Neurologic examination does not reveal any focal neurologic deficits or meningismus. Noncontrast and contrast-enhanced CT scans of the head are unremarkable. A lumbar puncture is performed and reveals an opening pressure of 35 mm Hg, WBC count 20 cells/mm3 with a lymphocyte predominance and glucose 40 mg/dL. The serum glucose is 120 mg/dL. The CSF gram stain is negative. Which of the following organisms is the most likely cause of his symptoms? Cryptococcus neoformans Cytomegalovirus Neisseria meningitidis Toxoplasma gondii

Correct Answer ( A ) Explanation: The patient's lumbar puncture reveals an elevated opening pressure, leukocytosis with lymphocyte predominance and a low glucose (< 60% of the serum level). This is consistent with a fungal meningitis. In the setting of acquired immunodeficiency syndrome (AIDS) and a CD4 count less than 100, the most likely cause of a fungal meningitis is Cryptococcus neoformans. This infection causes fever, headache, nausea and vomiting. It does not usually cause a significant inflammatory response, and, therefore, meningeal signs are often absent. The fungus is not visualized on Gram stain. The diagnosis can be confirmed with cryptococcal antigen level, India ink staining or a fungal culture. The treatment for cryptococcal meningitis is intravenous amphotericin B. After initial treatment, patients are started on chronic fluconazole to prevent recurrence.

Which of the following patients requires human rabies immune globulin? A person who was not previously vaccinated awakens to find a bat in his room and animal not captured A person who was previously vaccinated was bit by a dog and animal not captured A person who was previously vaccinated was bit by a fox and animal not captured A person who was previously vaccinated was bit by a rat and animal not captured A person who was previously vaccinated was bit by a squirrel and animal not captured

Correct Answer ( A ) Explanation: The question as to which patient requires human rabies immune globulin and human rabies vaccine after exposure to a potentially rabid animal is common in emergency medicine. Even though there was no clear bite or abrasion, the Advisory Committee on Immunization Practices (ACIP) recommends postexposure prophylaxis with immune globulin and vaccine for unvaccinated patients who have been in close proximity to bats, even if there is no sign of injury or damage to the skin, including waking up and finding a bat in the room. In an unvaccinated individual, this means human rabies vaccine and immune globulin given at the time of presentation.

A 26-year-old woman presents to the ED after finding a tick attached to her right flank. She believes it has been there since she went hiking four days prior. On exam, you notice a red annular rash on her right flank with mild central clearing. A urine beta-hCG test is positive. Her last menstrual period was six weeks prior to this visit. What antibiotic prescription should this patient receive? Amoxicillin, 14 days Doxycycline, 7 days Erythromycin, 7 days Trimethoprim-sulfamethoxazole, 14 days

Correct Answer ( A ) Explanation: This patient has a history and physical exam consistent with Lyme disease. Her rash is classic for erythema migrans, seen in 60%-80% of those with Lyme disease, usually in the first several days after a tick bite. This first stage of Lyme disease needs to be treated before it progresses to more serious symptoms affecting the neurological and cardiovascular systems. The presence of erythema migrans requires she be treated with a 14-day course of amoxicillin.

About two weeks after a hiking trip to South America, a 25-year-old man develops an abrupt onset of fever, abdominal cramping, and voluminous bloody diarrhea. A number of his fellow travelers who ate at a particular restaurant with him have the same symptoms. He also complains of severe right upper quadrant abdominal pain. A bedside ultrasound performed in the emergency department shows a round hypoechoic structure in the liver. He is unable to keep down food or water and appears severely dehydrated. What is the next best step in management? Admission for antimicrobials Attempted aspiration of the lesion identified on ultrasound exam Computed tomography scan of the abdomen Surgical consultation

Correct Answer ( A ) Explanation: This patient is suffering from dysentery caused, most likely, by a foodborne infection with Entamoeba histolytica. The most common presentation of amebic colitis is a gradual onset of bloody diarrhea, abdominal pain, and tenderness spanning several weeks. It has the capacity to invade the bloodstream and cause extraintestinal abscesses, most commonly in the liver. Amebic liver abscess may manifest years after travel to or residency in an endemic area. Microscopic stool examination for trophozoites from a single stool sample in amebic colitis is only 33%-50% sensitive. Examination of three stool samples over no more than 10 days can improve the detection rate to 85%-95%. Individuals with amebiasis should be treated with an antimicrobial. Diagnosis via aspiration of the abscess's contents (B) can be done, but it is invasive and not typically necessary for successful treatment. Antibiotics are usually sufficient to cure the abscess as well as the dysentery. Abdominal CT scan (C) may give additional information about the abscess but is not necessary in the ED, and it exposes the patient to unnecessary radiation. Surgical consultation (D) is not necessary at this time, especially when a course of antimicrobials has not yet been attempted.

24-year-old sexually active man presents with complaints of the "flu" for the past three weeks. Specifically, he complains of generalized weakness, malaise, myalgias, a low-grade fever, and anorexia. He denies URI symptoms or a cough. He has intercourse with men and does not always use condoms. He is concerned that he may have contracted HIV. His vital signs are significant for a blood pressure of 128/62 mm Hg, a heart rate of 82 beats per minute, an oxygen saturation of 99% on room air, and a temperature of 101.3°F (38.5°C). Your physical exam reveals a well-appearing male who is in no acute distress with no thrush or abnormal findings. Laboratory values reveal WBC of 6.2 with 3% bands, hemoglobin of 15.6 g/dL, and platelets of 120/microL. Chest radiograph and urinalysis are normal. Which of the following statements best describes the next management step? You suspect acute HIV syndrome, obtain an oral swab for rapid HIV testing, and admit the patient for confirmatory studies if positive You suspect acute HIV syndrome, obtain an oral swab for rapid HIV testing, and confirm with blood specimen if positive You suspect acute HIV syndrome, obtain an oral swab for rapid HIV testing, and discharge and reassure the patient if negative You suspect acute HIV syndrome, obtain an oral swab for rapid HIV testing, and make the diagnosis of HIV if positive

Correct Answer ( B ) Explanation: Acute HIV syndrome (also known as acute seroconversion syndrome) may follow primary exposure by two to four weeks and cause nonspecific symptoms, including fever, chills, malaise, myalgias, pharyngitis, diarrhea, or other neurologic or immunologic complaints. These symptoms are generally mild with spontaneous resolution and are often mistaken for a benign viral illness. As a result, many patients do not seek medical care during this phase. When testing for HIV, positive oral swab results should be considered preliminary pending confirmation (required) with serum testing via HIV-1/HIV-2 differentiation immunoassay or Western blot.

Which of the following is true regarding the characteristic rash of chickenpox? Lesions appear over days and fade by the third day Lesions appear over days with multiple stages present at once Lesions appear over two to four weeks with multiple stages present at once Lesions appear over two to four weeks with one stage present at a time

Correct Answer ( B ) Explanation: Chickenpox is a highly contagious but generally benign and self-limited viral disease caused by the varicella-zoster virus (also known as human herpesvirus 3). The disease is characterized by the sudden onset of fever, malaise, and a pustular maculopapular rash that can occur anywhere on the skin or mucus membranes. The lesions then become vesiculated followed by scabbing over the course of three to four days before resolving. Skin lesions appear in crops with multiple lesions of various stages appearing on the skin at the same time. Uncomplicated infection is generally treated with supportive measures, including antipyretic, antipruritic, and pain control medications. Antivirals such as acyclovir, valacyclovir, and foscarnet may also be initiated in severe disease or immunosuppressed individuals. Parents should be cautioned to avoid giving their children aspirin or aspirin-containing medications due to the risk of developing Reye syndrome.

12-year-old boy presents to the Emergency Department with fever and a rash. He was at a friend's house playing outdoors four days prior. The patient has the finding shown above. Which of the following is the therapy of choice? Benzathine penicillin Doxycycline Miconazole Trimethoprim-sulfamethoxazole

Correct Answer ( B ) Explanation: Doxycycline 100 mg PO twice daily for three weeks is the therapy of choice for the treatment of Lyme disease. Lyme disease is a tick-borne illness caused by the spirochete Borrelia burgdorferi and is the most common vector-borne disease in the United States. It is transmitted by the Ixodes tick. The classic erythema migrans rash appears in the early stage of the disease as an erythematous, oval, bull's eye shaped lesion at the site of the tick bite. It is the most characteristic clinical manifestation of Lyme disease and is also diagnostic. Other findings may include nonspecific generalized symptoms (e.g. malaise), meningeal irritation, or gastrointestinal upset. Progression of the disease may lead to cardiac or neurologic manifestations, and some may go on to develop chronic arthritis or encephalitis.

Which of the following patients with Salmonella enteritis should receive antibiotics? 13-month-old with diarrhea and no signs of volume depletion 13-year-old boy with sickle cell disease 19-year-old man living in a college dorm 8-year-old boy with no medical problems

Correct Answer ( B ) Explanation: Empirical antibiotic therapy for Salmonella enteritis is not recommended for otherwise healthy patients without severe disease. However, it is recommended in certain groups including patients with sickle cell disease. Salmonella is the most common documented bacterial enteritis in the US. The bacteria is acquired from ingestion of contaminated food, typically beef and poultry. The disease presents with fever, loose, watery stool that may have blood or mucous in them and colicky abdominal pain. Empirical antibiotic therapy is typically not required in healthy patients. Antibiotics do not shorten the duration of the disease and may increase the length of time that a patient is a carrier of the bacteria. Antibiotics are recommended (although not proven to be effective) based on severity of disease, age, immune system competency and other comorbidities. Additionally, health care workers or other people who may spread the disease should be treated to eradicate the carrier state. Treatment options include ciprofloxacin, norfloxacin and azithromycin.

20-year-old woman presents to the Emergency Department with lower abdominal pain and vaginal discharge. She is sexually active with two partners and uses condoms most of the time. Physical examination reveals thin yellow cervical discharge without cervical motion tenderness, adnexal tenderness or masses. Which of the following is the recommended therapy for her condition? Acyclovir 400 mg PO three times daily for 10 days Ceftriaxone 250 mg IM plus azithromycin 1 g PO Doxycycline 100 mg PO twice daily for 14 days Metronidazole 500 mg PO twice daily for seven days

Correct Answer ( B ) Explanation: Gonorrhea is the second most common sexually transmitted infection after chlamydia. It is caused by the gram-negative diplococcus Neisseria gonorrhoeae. Patients may be asymptomatic or present with lower abdominal pain, dysuria, or mucopurulent cervicitis. Many infections coexist with chlamydial infections. Symptomatic men may report dysuria or purulent penile discharge. Occasionally, infection leads to acute prostatitis or epididymitis. Men who have sex with men may have mucopurulent anal discharge heralding a diagnosis of gonorrheal proctitis. Infection may be disseminated. Signs and symptoms of disseminated gonorrheal infection include pustular skin lesions on an erythematous base, fever, malaise, arthralgias or arthritis. Nonculture diagnostic methods, such as nucleic acid amplification testing, from cervical, vaginal, penile or urine samples are recommended. Treatment of gonorrhea includes dual therapy to also cover Chlamydia trachomatis. Ceftriaxone 250 mg IM plus azithromycin 1 g PO is an appropriate regimen that can be administered in the emergency department and does not require the patient to take daily medication after discharge. An alternative to ceftriaxone is cefixime 400 mg PO once, and an alternative to azithromycin is doxycycline 100 mg PO twice daily for seven days.

6-year-old boy with a past medical history of glucose 6-phosphate dehydrogenase deficiency presents to the Emergency Department for a cough associated with vomiting. His parents state that he has been coughing every day for the last week. He is unimmunized but otherwise healthy. He appears well on exam with a dry cough and occasional wheezes. His laboratory workup reveals a leukocytosis with lymphocytic predominance and a chest X-ray with peribronchial thickening. He was seen by his pediatrician three days ago and tested positive for pertussis. Which of the following is the most appropriate therapy at this time? Ampicillin-sulbactam Azithromycin Ertapenem Trimethoprim-sulfamethoxazole

Correct Answer ( B ) Explanation: Pertussis is a highly contagious acute respiratory infection caused by the gram-negative rod Bordetella pertussis. Infection is spread by respiratory droplets. Childhood vaccination and natural immunity do not confer lifelong immunity against pertussis. School-aged children are usually affected, but adults may be carriers. Three clinical phases exist: catarrhal, paroxysmal, and convalescent. The catarrhal phase, during which infectivity is highest, lasts one to two weeks and is characterized by cough, low-grade fever, rhinitis, and anorexia. The paroxysmal phase lasts two to six weeks and is characterized by defervescence, increased coughing, and paroxysms of coughing with inspiratory "whooping" and posttussive emesis. The convalescent phase may include a cough lasting several months. The diagnosis is often clinical but is commonly missed due to atypical presentations and misdiagnosis as bronchitis. A polymerase chain reaction of nasopharyngeal secretions, nasopharyngeal culture or serological antibodies may be used for definitive diagnosis. Diagnostic workup may reveal lymphocytosis and radiographic evidence of peribronchial thickening, atelectasis or pulmonary consolidation. Treatment is with azithromycin, although trimethoprim-sulfamethoxazole may be used in macrolide-intolerant patients. Treatment is best if started during the first week of the illness and may have limited utility after the catarrhal phase.

13-month-old boy presents with a rash that developed yesterday on his neck. Mom notes that he had a high fever for 3 days earlier in the week which resolved yesterday. The child has been a little fussier than usual, but has otherwise been acting normally. On examination, he is afebrile. A diffuse maculopapular rash is noted on his face, trunk and extremities. What is the likely cause of his symptoms? Coxsackie virus Human herpesvirus 6 Parainfluenza virus Parvovirus B19

Correct Answer ( B ) Explanation: Roseola infantum, or sixth disease, is a benign febrile illness affecting children ages 6 months to 3 years. Human herpesvirus 6 is the most common cause of roseola. The illness starts with abrupt onset of a high fever that lasts three to four days. During that time, the child may also have symptoms of malaise, conjunctival injection, inflammation of the tympanic membranes or mild respiratory symptoms but is generally nontoxic in appearance despite the high fever. A key feature of the illness is that the rash starts when the fever resolves. The lesions are discrete, pink, blanching macules or maculopapules that start on the neck and trunk before spreading to the extremities and face. Is is typically nonpruritic and lasts one to two days. No treatment is necessary.

A 19-year-old man presents with a painless ulcer on the shaft of his penis present for the last three days. Examination is also notable for bilateral non-tender inguinal adenopathy. He reports he is sexually active with men. What is the most appropriate course of action? Ceftriaxone IM Penicillin IM Rapid plasma reagen test Viral culture

Correct Answer ( B ) Explanation: Syphilis is caused by the spirochete Treponema pallidum. Transmission occurs during exposure of moist skin to an infected area as the organism does not survive on dry surfaces. Syphilis progresses through three stages of illness. Primary syphilis is characterized by the chancre shown in the picture above. The lesion occurs at the site of exposure and begins as an erythematous papule that ultimately ulcerates. The ulcer has raised edges, sharply demarcated borders and a clean base. Left untreated, the chancre resolves after 2-6 weeks. Bilateral painless inguinal adenopathy is sometimes present. Secondary syphilis develops 5-8 weeks after resolution of the chancre. Most commonly, this stage involves a diffuse total body rash beginning on the trunk and spreading distally, often involving the palms and soles. During this stage, condylomata lata may develop (broad-based papules in the perineal area). Syphilis then enters a latent period, which can last for years in immunocompetent patients and then manifest most commonly affecting the cardiac or nervous systems. The treatment of primary syphilis is penicillin G benzathine 2.4 million units IM.

A 29-year-old man with a history of HIV presents with shortness of breath and fever. He has a productive cough but denies hemoptysis. You obtain the chest radiograph seen above. Which of the following is true regarding the patient's diagnosis? Elevated LDH is common Spread is by the hematogenous route Steroids should be administered prior to antibiotics if the PaO2 is <80 Trimethoprim-sulfamethoxazole is the treatment

Correct Answer ( B ) Explanation: The chest radiograph demonstrates miliary tuberculosis (TB), or acute disseminated tuberculosis. The term miliary was first used to describe the pathologic lesions seen on radiography that appeared as small millet seeds. Miliary TB occurs when the host is unable to contain a recently acquired or a dormant TB infection. The condition was mostly seen in young children after primary infection but now is more common in the elderly and in persons infected with HIV. Spread of the mycobacteria occurs through the hematogenous route, which leads to the multisystem nature of miliary TB. Clinically, patients develop many of the similar signs and symptoms of active pulmonary TB—fever, weight loss, anorexia, and weakness. Hemoptysis is uncommon. The classic miliary pattern seen in the radiograph is present in approximately 50% of cases. Hyponatremia is sometimes seen from the development of SIADH. Mortality rates are higher than for other forms of TB, which is likely due to a delay in treatment.

An 18-year-old man presents with penile discharge after unprotected sex. A urine GC/Chlamydia test is ordered. Which of the following is an appropriate treatment regimen? Ceftriaxone 125 mg IM X 1 and azithromycin 1,000 mg PO X 1 Ceftriaxone 250 mg IM X 1 and azithromycin 1,000 mg PO X 1 Ciprofloxacin 500 mg BID X 7 days and azithromycin 1 000 mg PO X 1 Wait for the test results in order to tailor treatment

Correct Answer ( B ) Explanation: The patient has urethritis, which is likely caused by either Chlamydia trachomatis or Neisseria gonorrhea. Chlamydial infection and gonorrhea, the 2 most common nonulcerative sexually transmitted infections, also can cause vaginal discharge, especially in the setting of mucopurulent cervicitis, and both tend to cause urethral discharge in men. These 2 infections commonly occur concurrently, and clinical manifestations are clinically indistinguishable. Thus they should be simultaneously treated in all patients. Ceftriaxone 250 mg IM will treat the vast majority of N. gonorrhea strains and azithromycin 1,000 mg PO will treat C. trachomatis.

7-year-old boy presents with a fever, cough, runny nose, and conjunctivitis for 3 days followed by the above pictured rash that initially appeared on his face. Exam of his oral mucosa reveals pinpoint gray spots. Which of the following is the most likely diagnosis? Erythema infectiosum Measles Rocky Mountain spotted fever Rubella

Correct Answer ( B ) Explanation: This patient has a typical presentation for measles (rubeola). The measles virus is a respiratory tract pathogen that is highly communicable. After exposure, the virus incubates for 10-14 days. The patient then develops cough, coryza and conjunctivitis along with fever. 2-4 days later, the characteristic rash begins on the face and spreads down the trunk then fades in a head-to-toe direction. Additionally, Koplik's spots, pinpoint gray spots with surrounding red inflammation, develop on the buccal mucosa and are pathognomonic for measles. Treatment is supportive.

12-year-old boy presents with high fever, muscle and joint aches, and headache for two days. He states he just got back from a camping trip in North Carolina. His exam is unremarkable. Labs are normal except for platelets of 95,000 and serum sodium of 128. Which of the following is the most appropriate next step in management? Obtain a Babesia microti DNA PCR and administer atovaquone and azithromycin Obtain a Rickettsia rickettsii immunofluorescence assay and administer doxycycline Send Lyme disease titers and administer amoxicillin Supportive care

Correct Answer ( B ) Explanation: This patient is suffering from early Rocky Mountain spotted fever (RMSF) from Rickettsia rickettsii. RMSF is a tick-borne illness which is endemic to North, South, and Central America. Although RMSF cases have been reported throughout most of the contiguous United States, five states (North Carolina, Oklahoma, Arkansas, Tennessee, and Missouri) account for over 60% of all cases. Clinically, RMSF is characterized by high fevers, arthralgias, myalgias, and a petechial rash, which begins on the ankles and wrists and spreads centrally. Early in the disease, there is often no rash and some patients with confirmed RMSF never develop a rash. Thus, early RMSF presents similar to a non-specific viral illness. The clinician may be steered towards the diagnosis of a RMSF by the presence of thrombocytopenia and hyponatremia. R. rickettsii is an obligate intracellular bacteria transmitted to humans by the American dog tick (Dermacentor variabilis) and the Rocky Mountain wood tick (Dermacentor andersoni). RMSF can progress to involve the cardiopulmonary system (myocarditis, AV blocks, dysrhythmias, interstitial pneumonitis, pulmonary edema, etc.), nervous system (encephalomyelitis, meningitis, cerebral thrombovasculitis, etc.) and cutaneous (vasculitis, ecchymosis, and ulcerations). Ricketsial antibodies can be performed but indirect immunofluorescence assay (IFA) is the standard test for diagnosis as it has a high sensitivity (95%). Treatment is most effective when started early in the course of disease. Doxycycline 100 mg twice a day for 7-10 days is the most common therapy but chloramphenicol can be substituted in those with a severe reaction to tetracycline antibiotics.

34-year-old man presents with a deep laceration to the left leg after falling off a motorcycle. The wound is contaminated with rocks and dirt. He states that he has not received a tetanus shot since completing vaccinations as a child. After irrigation and repair, which of the following should be administered? No tetanus prophylaxis needed Tdap Tdap and Tetanus immune globulin Tetanus immune globulin

Correct Answer ( B ) Explanation: This patient presents with a dirty wound and no tetanus booster in the last 5 years and thus requires a tetanus booster during this presentation. Tetanus is a toxin-mediated disease that is characterized by uncontrollable skeletal muscle spasms. It can cause hypoventilation, hypoxia and death if the toxin affects the muscles of respiration. Tetanus is a relatively rare disease particularly in developed countries where vaccination programs have been successful. Primary immunization confers protective antibodies to nearly 100% of patients. Immunity wanes between 5 and 10 years after completion of the initial vaccination series and so patients should have a booster shot every 10 years. It typically affects patients who have sustained a deep, penetrating wound. In adult patients with a history of a primary series in the past, any wound that is not a clean/minor wound should be given a tetanus shot (Tdap) if their last booster was >5 years ago. If the patient has a history of not completing a primary series, they should be given a Tdap regardless of the wound depth or size in addition to tetanus immune globulin if the wound is high risk.

A 33-year-old cattle farmer presets with a rash to his lower left arm as seen above. What treatment is indicated? Cephalexin Ciprofloxacin Topical bacitracin Topical corticosteroids

Correct Answer ( B ) Explanation: This patient presents with cutaneous anthrax requiring treatment with ciprofloxacin. Anthrax is caused by the bacteria Bacillus anthracis, a gram-positive, spore forming bacteria, and can manifest in a number of forms including cutaneous, gastrointestinal and pulmonary. Cutaneous anthrax commonly presents in animal product laborers but can also be the result of exposure to weaponized anthrax. Initially, the disease manifests as a small papule at the site of inoculation. This papule becomes erythematous with a central vesicle of bulla. This bulla then transforms into a hemorrhagic and necrotic lesion usually seen as a central black eschar. There is also regional adenitis accompanying the cutaneous lesion. The mortality associated with untreated cutaneous anthrax is high; roughly 20%. However, treatment reduces the mortality to 1%. Initiation of antibiotics does not change the local course of disease but prevents dissemination and death. Ciprofloxacin is the appropriate treatment for this form of the disease. An alternative to ciprofloxacin in cases of uncomplicated cutaneous anthrax is doxycycline.

A patient with AIDS presents with fever, severe headache, and positive Kernig's sign. CSF findings reveal lymphocytosis and increased protein levels. India ink staining reveals encapsulated yeast forms. Which of the following is the most likely pathogen? Borrelia burgdorferi Candida albicans Cryptococcus neoformans Neisseria meningitidis

Correct Answer ( C ) Explanation: Although all of the above pathogens can cause meningitis in a patient with AIDS, Cryptococcus neoformans has the following spinal fluid findings, including increased opening pressure, variable pleocytosis, increased protein, and decreased glucose. Gram stain of the cerebrospinal fluid usually reveals budding, encapsulated fungal cells. Prior to waiting on Gram-stain results, the CSF can be sent for detection of the cryptococcal antigen (highest sensitivity in AIDS patients). Detecting a serum cryptococcal antigen also aids in the diagnosis. Treatment of cryptococcal meningitis involves antifungal therapy such as amphotericin B and flucytosine, followed by fluconazole suppression until adequate immune reconstitution, or if this does not occur, indefinitely

Which of the following sexually transmitted infections has a high rate of cotransmission with HIV infection? Gardnerella vaginalis Neisseria gonorrhoeae Treponema pallidum Trichomonas vaginalis

Correct Answer ( C ) Explanation: Disorders characterized by genital ulcers (syphilis, herpes, chancroid, lymphogranuloma venereum, granuloma inguinale) have high rates of HIV cotransmission. Treponema pallidum is the causative agent of syphilis and is associated with a painless ulcer during the initial stage of infection (primary syphilis). Ulcers typically appear 3 to 6 weeks after exposure.

50-year-old woman presents to the emergency department with a rash that has progressively been spreading over the last several days. She indicates she was visiting her sister in Connecticut two weeks ago and she remembers suffering many insect bites. You diagnosis the rash as erythema migrans. What is the appropriate treatment for this patient? Azithromycin 500 mg orally daily X 7 days Cephalexin 500 mg orally four times a day X 7 days Doxycycline 100 mg orally twice a day X 21 days Rifampin 300 mg orally twice a day X 60 days

Correct Answer ( C ) Explanation: Lyme disease is the most common tick-borne illness in the United States and is caused by the spirochete Borrelia burgdorferi. Early localized Lyme disease is manifested as erythema migrans, which usually occurs within one month of a bite from the Ixodes tick. Several randomized-controlled trials have been conducted to evaluate the most effective treatment for early localized Lyme disease. Based upon the results, oral doxycycline, amoxicillin and cefuroxime have equal efficacy for the treatment of Lyme disease. Doxycycline is often the choice of treatment because it is also effective in treating the co-infecting agent, Anaplasma phagocytophilum. Additionally, doxycycline has better central nervous system penetration.

A 37-year-old man with a history of human immunodeficiency virus with a previous CD4+ count of 90 cells/µL presents to the emergency department feeling poorly with a mild headache, neck pain with stiffness and nausea. He has not been compliant with his medications. Physical examination shows vital signs of HR 97, BP 133/70, RR 16, T 99.0℉. The patient has noticeable meningismus. Head CT performed is unremarkable. Lumbar puncture is performed during the workup. Opening pressure is 40 cm H2O and results show WBC 75 cells/µL with 20% PMNs, glucose 24 mg/dL, protein 290 mg/dL and negative gram stain. What further cerebrospinal fluid analysis should be performed to confirm the most likely diagnosis in this patient? Acid-fast bacilli test Herpes simplex virus PCR India ink stain Venereal Disease Research Laboratory test

Correct Answer ( C ) Explanation: Patients who are immunocompromised with entities such as AIDS are at increased risk for serious infections including meningitis. This is particularly true in patients with a CD4+ count less than 200 cells/µL. Cryptococcus neoformans is the most common cause of meningitis in patients with AIDS and ultimately affects 10% of patients with AIDS. As with other opportunistic infections, HAART has dramatically decreased the incidence of cryptococcal meningitis. However, it remains a leading cause of mortality for people with HIV infection. It usually affects patients with a CD4+ count below 100 cells/µL. A lumbar puncture is diagnostic; elevated opening pressures are typical and evidence of encapsulated yeast forms on staining of the cerebrospinal fluid with India ink can be found. Other CSF findings include elevated protein, low glucose and moderately elevated WBC but less than 500 cells/µL. Cryptococcus neoformans Patient will be HIV (+) Complaining of headache, fever, stiff neck, photophobia, vomiting Labs will show CD4 < 100 Diagnosis is made by India ink stain of CSF (round encapsulated yeast), Cryptococcal antigen (CrAg) - CSF or serum Treatment is amphotericin B (fungicidal), flucytosine (fungicidal), fluconazole (fungistatic)

Which of the following findings on history or physical examination is most consistent with Rocky Mountain spotted fever? Ascending flaccid paralysis Biphasic fever Centripetal rash Lymphadenopathy

Correct Answer ( C ) Explanation: Rocky Mountain spotted fever is a tick-borne illness caused by infection with Rickettsia rickettsii. The disease is most prevalent in the southeastern United States and is transmitted primarily by the American dog tick (Dermacentor variabilis) and the Rocky Mountain wood tick (Dermacentor andersoni). After introduction into the host, R. rickettsii invades the vascular endothelial cells which leads to direct damage of the cells, increased vascular permeability and activation of clotting factors. The host response to the infection also plays a role in the clinical manifestations of the disease. The vast majority of patients present with a fever and rash. The rash typically starts on the third to fifth day of the illness as a blanching erythematous macular rash that starts on the wrist and ankles. It then spreads centripetally to involve the extremities and trunk. The rash becomes maculopapular and, eventually, petechial in appearance. The lesions may occasional coalesce to form large ecchymotic areas.

55-year-old man with no medical history and not on any medications presents to the ED complaining of irritability, generalized weakness, myalgias, muscle cramps, and difficulty opening his mouth. On exam, his vital signs are within normal limits. As you are examining the patient, he begins to experience body movements seen in the video above. During this time, the patient is awake and able to answer questions. Which of the following is true regarding his diagnosis? Diphenhydramine is the recommended treatment for his condition It is caused by the inhibition of acetylcholine release resulting in neuromuscular blockade Minor stimuli such as touch or noise can worsen the condition The incidence has been rising over the past 25 years

Correct Answer ( C ) Explanation: Tetanus is a toxin-mediated disease characterized by severe uncontrolled skeletal muscle spasms. It is caused by the bacterium Clostridium tetani. The most common portals of entry for the organism are puncture wounds, lacerations, and abrasions. C. tetani produces the neurotoxin tetanospasmin (TS) at the site of tissue injury. TS binds preferentially to inhibitory GABAand glycinergic neurons and blocks presynaptic release of these neurotransmitters. Without inhibitory control, the motor neurons undergo sustained excitatory discharge, resulting in muscle spasm characteristic of tetanus. The classic initial presenting symptom of trismus(lockjaw) caused by masseter spasm is present in 50%75% of patients. As the other facial muscles become involved, a characteristic sardonic smile (risus sardonicus) appears. As the disease progresses, generalized uncontrollable muscle spasms can occur spontaneously or as a result of minor stimuli such as touch or noise. Spasms may result in vertebral and long-bone fractures and tendon rupture. Opisthotonus is a prolonged tonic contraction that closely resembles decorticate posturing. Spasms of laryngeal and respiratory muscles can lead to ventilatory failure and death. Treatment involves avoidance of unnecessary stimulation, benzodiazepines, and antibiotics (metronidazole).

Which of the following meets criteria for a diagnosis of systemic inflammatory response syndrome (SIRS)? Heart rate > 80 and respiratory rate > 16 Heart rate > 90 Heart rate > 90 and temperature > 100.4°F Temperature > 100.4°F and WBC > 10,000

Correct Answer ( C ) Explanation: The SIRS criteria is part of the sepsis syndrome spectrum of diseases. SIRS often represents the body's host response to an infection. Although research has found that SIRS criteria alone does not predict an increased mortality, it should prompt continued investigation for an underlying pathology. The presence of organ dysfunction and shock, however are significant predictors of adverse outcomes and should be fully addressed. Sepsis is the tenth most common cause of death in the US.

20-year-old otherwise healthy woman presents to the ED with bloody diarrhea three days after eating a hamburger at a local fast food restaurant. Her diarrhea started yesterday, but she became concerned after she noticed a small amount of blood in her stool earlier today. Vital signs include temperature 37.8°C and heart rate 102 bpm. She has no abdominal tenderness with palpation. Which of the following is the best next step in management? Azithromycin Ciprofloxacin Intravenous fluids Loperamide

Correct Answer ( C ) Explanation: The treatment for most causes of food poisoning or foodborne gastroenteritis should focus on rehydration with intravenous fluids. Campylobacter and Salmonella are two of the more common causes of food-related bacterial enteritis in developed countries, and both are self-limiting. Campylobacter enteritis is caused by consumption of undercooked poultry, contaminated beef, pork, or raw milk. Diarrhea (often bloody), abdominal pain, fever, and dysentery usually develop within two to three days of ingestion of contaminated produce. Salmonella is most commonly caused by ingestion of contaminated poultry or beef products. Other causes include unpasteurized milk, eggs, or fish. Symptoms are similar to Campylobacter enteritis but usually develop within 24 hours. The main treatment is supportive, but antibiotics may be indicated in patients who are immunocompromised, older than 50 years old, younger than 3 months, or have severe colitis.

Which of the following is the most common laboratory abnormality seen in ehrlichiosis? Elevated serum creatinine Hyponatremia Leukopenia Thrombocytosis

Correct Answer ( C ) Explanation: There are two forms of human ehrlichiosis in the United States - human monocytic ehrlichiosis caused by Ehrlichia chaffeensis and human granulocytic anaplasmosis caused by Anaplasma phagocytophilum. Both illnesses are tick borne and have similar presentations and clinical findings. The incidence of illness seems to be increasing with most cases occurring in the spring and summer months. Ehrlichia are gram-negative, obligate intracellular rickettsia-like coccobacilli. The main two vectors of infection are the Lone Star tick (Amblyomma americana) and the Ixodes tick which also transmits Lyme disease and babeiosis. The majority of patients provide a history of a tick bite with an average time between bite and symptom onset of nine days. Patients presents with abrupt onset of fever, chills, malaise, headache and myalgias. Gastrointestinal symptoms, such as nausea, vomiting and diarrhea, are less common. In contrast to other tick-borne illnesses such as Lyme disease and Rocky Mountain spotted fever, only one-third of patients develop a rash, which can be macular, maculopapular or petechial. The most common laboratory abnormalities, found in up to 90% of patients, include leukopenia, thrombocytopenia, and elevated transaminases. Diagnosis is primarily based on clinical presentation. Treatment consists of a ten day course of doxycycline.

A 15-year-old girl presents with fever after returning from a trip to Africa. She states that she has been getting fevers every three days for the last 10 days. What test will confirm a diagnosis of malaria? Complete blood count Enzyme linked Immunosorbent assay (ELISA) Thick and thin smears VDRL

Correct Answer ( C ) Explanation: Thick and thin smears of a patient's blood sample are the gold standard for the diagnosis of malaria. Malaria is an acute febrile illness caused by 4 major types of Plasmodium: falciparum, ovale, vivax and malariae. The disease is transmitted to humans via the female Anopheles mosquito. Plasmodium sporozoites from Anopheles saliva invade and multiply in hepatic cells. Lysis of the hepatic cells leads to release of the parasite into the blood stream. The parasite subsequently invades red blood cells (RBCs) and feeds on hemoglobin. RBC lysis follows and is accompanied by fever. In addition to fever, patients often present with headache, nausea, chills, fatigue, abdominal pain and anemia. Unlike the other strains, P. falciparum can cause organ dysfunction and death. Although the history and physical with particular focus on travel history to endemic malaria regions suggests the diagnosis, the gold standard test is microscopic evaluation of a thick and thin smear of blood. Peripheral blood is stained with Giemsa or Wright stains that reveal the presence of the parasite.

An 18-year-old college student with a history of HIV (CD4+ 250) presents to the ED with headache, fever, and stiff neck for two days. He thought he had a cold and has been taking acetaminophen without relief of his headache. Vital signs are T 102.38°F (39.1°C), BP 100/50 mm Hg, HR 140 bpm, and RR 30/min. He is sleepy but arousable. On exam, you place the patient's right hip and knee into a flexed position and then proceed to extend the knee. The patient winces when the knee is just beyond 90 degrees of flexion. You also note petechiae on his trunk and extremities with one small area on his right forearm that looks like a purple patch with a gray necrotic center. Which of the following is the most likely diagnosis? Cryptococcal meningitis Herpes encephalitis Meningococcemia Pneumococcal meningitis Toxoplasmosis

Correct Answer ( C ) Explanation: This patient has meningococcemia, a disease caused by Neisseria meningitidis. The clinical presentation ranges from a mild febrile illness to fulminant disease progressing to death within hours. Patients with meningococcal meningitis may present similarly to patients with meningitis of other origins with headache, photophobia, vomiting, fever, and signs of meningeal inflammation. Petechiae generally appear on the extremities and may progress to involve almost any body surface. Macular lesions may progress to purpura and ecchymoses in fulminant meningococcemia (purpura fulminans). The patient in this scenario exhibits a positive Kernig sign, representing meningeal irritation, and has a purpuric lesion on his right forearm characterized by a gray necrotic center surrounded by a purple ring. Morbidity and mortality are high in meningococcemia but reduced with prompt recognition and immediate initiation of antibiotic therapy. Ceftriaxone and vancomycin are acceptable first-line agents. Patients with HIV are prone to the same infections as a non-HIV infected individual, in addition to opportunistic infections. The best predictor of immunologic susceptibility to opportunistic infection is the CD4 cell count, with counts below 200 associated with increased risk. Cryptococcal meningitis (A) occurs in 10% of patients with HIV infection, but most commonly in those with CD4 cell counts less than 100 cell/µL. Headache and fever commonly occur, but there are no cutaneous manifestations. Treatment includes amphotericin B plus 5-flucytosine. HSV encephalitis (B) is associated with fever, headache, and focal neurologic signs, often localized to the temporal lobe. The patient may complain of a bad odor not perceived by anyone else (temporal lobe hallucination). Treatment includes acyclovir. Pneumococcal meningitis (D) is caused by Streptococcus pneumoniae, the most common cause of meningitis in adults. Symptoms are similar to meningococcal meningitis but do not progress to purpura fulminans. Treatment is also similar with ceftriaxone and vancomycin. Toxoplasma gondii (E) infection is the most common cause of focal intracranial mass lesions in patients with HIV infection. It is associated with fever, headache, and seizures. It is not associated with skin lesions. Treatment includes pyrimethamine plus sulfadiazine.

22-year-old landscaper presents to the ED with a rash that began approximately three weeks ago. He reports it started with a single bump on his left forearm that has been oozing fluid. He subsequently developed lesions tracking up his arm. He denies any joint pain, decreased range of movement, fever, numbness, or weakness. On exam, he has an area of redness overlying a small papule and a rash spreading proximally as depicted in the image above. Which of the following is the most appropriate treatment for this condition? Acyclovir Hydrocortisone 1% Itraconazole Permethrin

Correct Answer ( C ) Explanation: This patient has sporotrichosis, a fungal infection transmitted by inoculation into the skin with the fungus Sporothrix schenckii. It is most commonly seen in gardeners, farmers, landscapers, and other agricultural workers. The infection is classically associated with a seemingly innocuous puncture wound from a rose bush thorn. Its characteristic rash begins with an ulcer or papule and then spreads proximally along lymphatic channels with skip lesions. Lymphocutaneous involvement, as depicted in the picture, is the most common manifestation. Treatment for this is three to six months of itraconazole.

37-year-old man presents to the ED after he crashed his dirt bike going over a jump. You note a 5 cm wound to his right leg with a significant amount of dirt and debris. He does not think he has had a tetanus vaccine in the past as his mother is against vaccines. Which of the following is the most appropriate management? Tetanus immunoglobulin Tetanus immunoglobulin and metronidazole Tetanus immunoglobulin and tetanus vaccine Tetanus vaccine

Correct Answer ( C ) Explanation: This patient is at risk for tetanus. Tetanus is caused by Clostridium tetani, which is an anaerobic, motile, spore-forming, Gram-positive rod that is found in soil, dust, and feces and is resistant to heat and chemicals. Greater than 70% of cases of tetanus are caused by wounds, including post-operative infections. The incubation time for tetanus is generally three days to two weeks. Signs and symptoms of tetanus include weakness, myalgias, dysphagia, hydrophobia, and drooling. Patients may also exhibit trismus, risus sardonicus (facial muscle spasms), and generalized severe muscular spasms. Death is commonly secondary to laryngeal or respiratory spasm leading to respiratory failure and arrest. Patients may also exhibit autonomic dysfunction. There is no effect on mental status. Tetanus is a clinical diagnosis. Management of tetanus is first supportive and includes minimizing stimuli and administering benzodiazepines and narcotics for the spasms. Intubation and nondepolarizing neuromuscular blockade may be necessary. These patients should be admitted to an ICU for further care. To eliminate toxin production, the wound should be cleaned and debrided and metronidazole given. Additionally, tetanus immunoglobulin (TIG) should also be given as this may shorten the course and reduce the severity of the disease. TIG is safe in pregnancy. TIG is not needed if the primary series of vaccines was completed (3 or more tetanus toxoid vaccinations). Tetanus immunization should be considered for all wounds in the ED. If the wound is dirty, administer Td if > 5 years since last booster. If the wound is clean, administer Td if > 10 years since last booster. If the patient is allergic to Td, administer TIG and consider prophylactic antibiotics. Tetanus immunoglobulin is needed for dirty wounds as well if the patient did not complete the primary series of vaccines. Tetanus immunoglobulin (A) should be given as it is a dirty wound and the patient is not vaccinated; however, he also needs a diploid tetanus vaccine given that he has not been vaccinated previously. Tetanus immunoglobulin and metronidazole (B) would be indicated if the patient had signs and symptoms of active tetanus in order to neutralize the toxin as well as treat the disease process. Tetanus vaccine (D) should be given as he has not been vaccinated previously and has a dirty wound. He should also be given TIG for the same reasons in combination with the diploid vaccine.

A 27-year-old pet shop owner presents to the ED with a lesion on the back of her right hand. She describes a red, raised bump that has grown larger and into a purple bump over the course of several weeks. On exam, you note the multiple purple nodules along a lymphatic route. What is the most likely cause of her skin lesion? Mycobacterium avium intracellulare Mycobacterium kansasii Mycobacterium marinum Mycobacterium ulcerans

Correct Answer ( C ) Explanation: This patient most likely has "fish tank granuloma," a persistent granulomatous infection caused by the handling of tropical fish tanks or other activities involving contact with fish. Mycobacterium marinum is the infectious agent. Exposure is usually through an abrasion or break in the skin. It is characteristically associated with a skin lesion that develops three weeks after exposure. The lesion begins as a red plaque that progresses into a nodule or cluster of nodules along lymphatics. Lymphadenopathy is uncommon. Treatment can be difficult and may require several months of antibiotic therapy.

63-year-old patient who is currently undergoing chemotherapy for breast cancer presents after being sent in by her oncologist for positive blood cultures. The blood cultures showed gram positive cocci in clusters in two bottles. She had a fever two days ago when the cultures were drawn. Other than generalized fatigue and mild nausea, she has been asymptomatic. What is the best choice for empiric management? Cefepime Nafcillin Piperacillin tazobactam Vancomycin

Correct Answer ( D ) Explanation: A gram stain showing gram positive cocci in clusters often represents infection with Staphylococcus species, most notably S. aureus. Bacteremia may result as a complication of a primary infection, such as cellulitis or pneumonia, or may be the result of an infected central venous catheter. A careful history and physical examination is necessary to uncover clues as to the possible source of infection. Particular care should be directed at evaluating for possible endocarditis which may present with protracted fever and chills, new-onset murmur, Janeway lesions, Osler nodes and Roth spots. In this patient, given her history of current chemotherapy treatment for breast cancer, bacteremia due to the presence of a central venous catheter is the likely cause. Empiric treatment should include coverage of methicillin-resistant S. aureus (MRSA) until susceptibility is available. Vancomycin (15-20 mg/kg/dose every 8-12 hours, not to exceed 2 grams per dose) is the antibiotic of choice. Daptomycin is an acceptable alternative. Once susceptibility results are available, antibiotics can be adjusted accordingly. Cefepime (A) is a fourth-generation cephalosporin that provides gram-negative and some gram-positive coverage. While active against P. aeruginosa, it is does not provide coverage against MRSA. Nafcillin (B) is appropriate treatment for methicillin-sensitive S. aureus bacteremia. Given the risk of MRSA, it should not be used for empiric therapy prior to susceptibility results. Piperacillin tazobactam (C) is a broad-spectrum antibiotic but also does not provide coverage against MRSA.

A 37-year-old woman with a past medical history of acquired immunodeficiency syndrome presents with two days of fever and chills. She currently takes no medications. Which of the following is an indication for initiating antiretroviral therapy regardless of CD4+ cell count? Absolute neutrophil count < 1500 cells/mm3 Fever and cough Hepatitis A co-infection Pregnancy

Correct Answer ( D ) Explanation: AIDS is currently defined as the presence of an AIDS-defining illness or a CD4+ T cell count < 200 cells/mm3. Patients with CD4+ T cell counts < 200 cells/mm3 or viral load > 50,000 copies/mm3 are at increased risk for developing an AIDS-defining illness. These include Pneumocystis pneumonia, CMV retinitis, Kaposi's sarcoma, or infection with opportunistic organisms such as Mycobacterium avium complex, Coccidioidomycosis, Cryptococcosis, or Cryptosporidiosis. Antiretroviral therapy (ART) should be initiated in patients infected with human immunodeficiency virus (HIV) with a CD4+ T cell count < 350 cells/mm3 or an AIDS-defining illness. Current recommended initial therapy includes two nucleoside reverse transcriptase inhibitors (e.g., zidovudine, didanosine) plus one of the following: a nonnucleoside reverse transcriptase inhibitor (e.g. efavirenz), a protease inhibitor (e.g. indinavir), or an integrase inhibitor (e.g. raltegravir). Indications for initiation of therapy regardless of CD4+ T cell count include pregnancy, co-infection with hepatitis B virus and HIV-associated nephropathy.

45-year-old man presents to the emergency department worried about a tick bite. He indicates he just arrived home from Connecticut and on the airplane he noticed a tick on his leg. He denies any symptoms. He denies hiking and camping. He was visiting his grandmother and only notes that he was outside gardening with her two days ago. He is very concerned about Lyme disease because he has read about it in the newspaper. Upon examination you find what appears to be an engorged Ixodes tick attached to the patient's lower left leg. No rash is noticeable. What is the most appropriate management of this tick bite? Carefully remove the tick and discharge Carefully remove the tick and perform serologic IgM and IgG antibody testing for Borrelia burgdorferi Carefully remove the tick and send it for testing to determine if it is infected with Borrelia burgdorferi Carefully remove the tick and treat with doxycycline before discharge

Correct Answer ( D ) Explanation: Lyme disease is the most common tick-borne illness in the United States and is caused by the spirochete Borrelia burgdorferi. Individuals at highest risk for Lyme disease are those who live in endemic regions and have occupational or recreational exposure to ticks. However, in highly endemic regions people may even be at risk in their yards. The factors affecting transmission of Lyme disease include whether the tick was of the Ixodes species, if the tick attached, how long the tick was attached and if it was engorged. Ticks found walking on the skin or ticks that are easily removed are not attached and hence will not transmit disease. The duration of tick attachment is important because B. burgdorferi are rarely transmitted within the first 48 hours of attachment. If a tick is bigger or engorged then that means it has been feeding and is more likely to have transmitted disease. When a tick is detected, the first step is carefully removing the tick without squeezing or crushing it so as to not spill contents of the tick into the wound. The Infectious Disease Society of America recommends antibiotic prophylaxis if patients meet ALL of the following criteria: The attached tick is identified as an Ixodes scapularis tick; the tick is estimated to have been attached for greater than 36 hours (based on known exposure or degree of engorgement); prophylaxis is begun within 72 hours of tick removal; greater than 20% local rate of infection of ticks with B. burgdorferi (part of New England and mid Atlantic states, Wisconsin and Minnesota); and doxycycline is not contraindicated. Based on the degree of engorgement by an Ixodes tick, and considering the high rate of infection of ticks in New York, the patient in this question meets these criteria and should be treated appropriately with doxycycline.

7-year-old girl presents with the rash seen above. Her mother states it appeared seven days after hiking through the woods near their home in New Jersey. She has no known drug allergies. Which of the following is the most appropriate treatment for this patient? Ampicillin Azithromycin Cephalexin Doxycycline

Correct Answer ( D ) Explanation: Lyme disease is the most common vectorborne disease in the United States. It is a tickborne illness caused by the spirochete Borrelia burgdorferi. The early phase of the disease results in the characteristic rash seen above, erythema migrans. Prompt treatment of early disease can shorten the duration of symptoms and prevent progression to later stages of disease. Previously it was recommended that pregnant or lactating women and children younger than 8 years of age should receive amoxicillin. This is still a viable alternative in patients that cannot tolerate doxycycline. Doxycycline, a tetracyline, may be used for all patient populations and can be recommended if the treatment length is less than 21 days in pregnant patients and children. Advanced or severe disease should be treated with intravenous ceftriaxone or penicillin.

19-year-old man, who is a college student, presents to the ED with concern for a lesion on his penis for the past two days. He began a relationship with a new sexual partner three weeks ago. On exam, there is a nontender 1 cm ulcer on the dorsum of his glans. There is no inguinal adenopathy. An HIV ELISA and RPR are negative. What is the most likely diagnosis? Chancroid Granuloma inguinale Lymphogranuloma venereum Primary syphilis

Correct Answer ( D ) Explanation: Primary syphilis is characterized by a small papule that develops at the site of inoculation (usually genital) that becomes a painless, indurated ulcer, often described as the classic chancre. The chancre develops after an incubation period of 10-90 days, is present for 3-6 weeks, and resolves spontaneously. Serologic tests (VDRL and RPR) can be falsely negative for up to four weeks after the chancre appears and should not be relied upon to rule out primary syphilis. They are, however, quite sensitive for ruling out the diagnosis in later stages.

A 17-year-old woman presents with a fever, myalgias, and headache. She noted a rash that began 4 days after she noted her fever. The macular rash began on her wrists and ankles and spread toward her chest. She recently returned from hiking the Appalachian Trail in North Carolina about 1 week ago and is concerned about her symptoms. Which of the following is the most likely diagnosis? Babesiosis Colorado tick fever Lyme disease Rocky Mountain spotted fever

Correct Answer ( D ) Explanation: Rocky Mountain spotted fever (RMSF) is a life-threatening infection caused by Rickettsia rickettsia transmitted by dog ticks. Its name derives from its original description in Montana and Idaho in the late 19th century and from the typical petechial rash occurring initially on the wrists and ankles and spreading centripetally (towards the center). It may also involve the palms and soles. Despite its name, most cases are reported from the southeastern and southcentral United States. A little more than 50% of the cases present with the classic triad of rash, fever, and tick exposure, although the rash is rarely present during the first 3 days of the illness and usually appears on Day 4. Abrupt onset of fever, severe headache, and myalgias are the most common presenting symptoms 5-7 days after the tick bite.

21-year-old man presents with a macular rash on his palms and soles. He recalls a red mark on his penis last month but did not seek evaluation. Which is the most appropriate treatment? Azithromycin Ceftriaxone Ciprofloxacin Penicillin

Correct Answer ( D ) Explanation: Syphilis is caused by the spirochete Treponema pallidum. This patient's clinical presentation is classic, but syphilis has been called the "great imitator" because of Treponema's ability to infect almost any organ in the body. Secondary syphilis manifested by a total body macular rash classically involving the palms and soles occurs 5 to 8 weeks after the primary genital ulcer. A single dose of long-acting penicillin G benzathine administered intramuscularly is the treatment for primary and secondary syphilis.

72-year-old man from a nursing home presents with altered mental status. His rectal temperature is 38.4°C. Which of the following is an additional criterion for systemic inflammatory response syndrome (SIRS)? Heart rate 88 Lactic acid 3.8 mg/dL Respiratory rate 20 White blood cell count 13,000 cells/mm3 with 76% neutrophils, 10% lymphocytes and 12% bands

Correct Answer ( D ) Explanation: The goal of sepsis care has evolved to include early recognition of sepsis and subsequent aggressive fluid resuscitation and antibiotic administration within one hour. The SIRS criteria were developed in an attempt to capture patients who exhibit abnormal vital signs and laboratory data secondary to an infection. Patients meet criteria for SIRS when two or more abnormalities are identified. When patients have SIRS plus an infection, they have sepsis. It is important to recognize that other conditions can cause the abnormalities of SIRS including trauma, hemorrhage, pancreatitis, burns and overdose. The definition of SIRS include:A heart rate of 88 (A) does not meet criteria for SIRS. A measured lactic acid of 3.8 mg/dL (B) is not part of the SIRS criteria. Lactic acid has become an important marker of increased risk of mortality in patients with sepsis. As sepsis evolves and patients suffer from tissue hypoperfusion, levels of lactic acid increase in the blood. Once a patient has an elevated lactic acid level above 4 mg/dL, it is considered severe sepsis. Severe sepsis is also marked by signs of end organ damage (e.g. impaired renal function, coagulopathy, abnormal mental status). With aggressive resuscitation, serial lactic acids are measured to ensure its clearance. If initial fluid resuscitative efforts have not led to a decrease in lactic acid by 10%, additional fluid is indicated. The respiratory rate must be greater than 20 (C) in order to meet SIRS criteria.

Which of the following tick-borne illnesses is most associated with skin ulcers and lymphadenopathy? Babesiosis Colorado tick fever Lyme disease Tularemia

Correct Answer ( D ) Explanation: The major tick-borne diseases include Rocky Mountain spotted fever (RMSF), Lyme disease, ehrlichiosis, babesiosis and tularemia. Tularemia is caused by Francisella tularensis and is transmitted to humans either by tick bites or handling of infected animals (rabbits and rodents). Each of these diseases begins with non-specific viral-syndrome like symptoms including fever, myalgias, arthralgias and headache. Clinicians may be alerted to the presence of a tick-borne illness if a history of exposure (hiking in the woods, tick bite) are elicited. However, up to 50% of patients with a tick-borne illness do not recall a tick bite. Additionally, if labs are obtained, there are some common findings that would differentiate from a viral illness. Thrombocytopenia is often seen in RMSF, ehrlichiosis, tularemia and babesiosis. A mild elevation in hepatic transaminases is associated with RMSF, Lyme disease, ehrlichiosis, tularemia and babesiosis. Tularemia is associated with ulceroglandular disease.

26-year-old woman presents two days after an operation for recurrent sinusitis. Her husband states that she has been confused since she got "the flu" yesterday. Her vitals are a temperature of 103.1°F (39.5°C), HR of 115 bpm, BP of 95/70 mm Hg, and oxygen saturation of 99% on room air. On exam, she is disoriented and has a diffusely hyperemic, blanching rash. She has a surgical dressing covering her nose. What is the next step in management? Administer broad-spectrum antibiotics Obtain a CT scan of the sinuses Perform a wound culture Remove the surgical dressing

Correct Answer ( D ) Explanation: The surgical dressing must be removed to ensure there is no nasal packing or other foreign body present that could serve as a precipitant of toxic shock syndrome (fever, hypotension, diffuse erythroderma, multisystem organ dysfunction). Toxic shock syndrome is commonly associated with postsurgical dressings as well as vaginal foreign bodies (classically, extended-use tampons). Toxic shock syndrome toxin-1 (TSST-1)-producing strains of Staphylococcus aureus cause the infection, with the toxin serving as a superantigen that leads to overstimulation of T lymphocytes and subsequent massive, unregulated cytokine release. Patients often report a prodrome of flu-like symptoms including headache, myalgias, vomiting, and diarrhea.

58-year-old man presents to the ED with complaints of severe fever and chills, a diffuse headache, abdominal cramping, nausea and vomiting, and general weakness with lethargy. He returned from a lengthy business trip to Central America six days ago and has been experiencing such symptoms every three days. He notes that he had some generalized irregular fever while in Central America, but did not seek help at that time. His blood pressure is 148/72 mm Hg, respiratory rate 24, heart rate 122, oxygen saturation of 96% on 2L NC, and temperature 40.6°C. On exam, you note scleral icterus. Physical exam is unremarkable, with guaiac negative stool. Lab results reveal a hemoglobin of 8 g/dL, platelets 175/µL, LDH 800 U/L, and indirect bilirubin 2.5 mg/dL. Which of the following is the most likely cause of this patient's symptoms? Babesiosis Chagas disease Dengue fever Malaria

Correct Answer ( D ) Explanation: There are four known types of malaria: Plasmodium falciparum, Plasmodium vivax, Plasmodium ovale, and Plasmodium malariae. Although P. vivax is the most common form, P. falciparum leads to the most virulent disease, causing most cases of severe malaria and most malaria-related deaths. Because of its ubiquitous presence in much of the world, malaria should be considered in any patient with severe fevers and chills who reports recent travel to an endemic region. Classically, malaria begins with a flu-like prodrome that progresses to cyclical episodes of chills followed by fever, each lasting about two hours. These episodes recur every three days with P. vivax and P. ovale and every four days for P. falciparum and P. malariae. These episodes are associated with a hemolytic anemia, as identified in this patient by scleral icterus, low hemoglobin, elevated LDH, and indirect bilirubin. Identifying Plasmodial parasites on Giemsa-stained thick and thin smears makes the definitive diagnosis.

14-year-old girl presents to the ED with agitation, hallucinations, and myalgias. Her father notes she has been sick recently with a viral upper respiratory illness. Upon attempting to drink water, she begins to choke and retch violently. To which of the following was she likely exposed? Bat feces Cyanide Organophosphates Skunk bite

Correct Answer ( D ) Explanation: This patient is exhibiting signs and symptoms of rabies. Rabies causes over 40,000 deaths per year worldwide; however, it is extremely uncommon in the United States. Rabies is a neurotrophic rhabdovirus that is transmitted through saliva into bite wounds or mucous membranes. The virus replicates in the muscle cells near the bite site and stays for an incubation period of 30 to 90 days. It then rapidly ascends along peripheral nerves to the CNS where it shows a predilection for the pons and medulla. It enters the salivary glands after replication. Signs and symptoms of rabies include a viral prodrome of headache, fever, rhinorrhea, sore throat, myalgias, and vomiting. Patients then begin to show signs of encephalitis including agitation, hallucinations, ataxia, weakness, and seizures. The agitation is often mistaken for psychiatric illness. Neurologic symptoms are then followed by aerophobia (fear of air in motion) and severe hydrophobia (fear of water). Hydrophobia manifests as violent diaphragmatic contraction while drinking water and a strong protective gag reflex. Coma occurs within one week and death follows shortly thereafter. Diagnosis of rabies is clinical and involves a good exposure history. The diagnosis may be confirmed by brain biopsy. There is no definitive effective treatment for clinical rabies. Postexposure prophylaxis is key in preventing clinical disease. Prophylaxis should be undertaken for any high risk bites such as those from raccoons, skunks, foxes, bats, or coyotes in the United States. It should be done even for seemingly insignificant exposures to bats in endemic regions including sleeping in a room where a bat was found. Domestic animals generally pose low risk and the animal may be observed for ten days for the development of symptoms before postexposure prophylaxis is administered given the long latent period. Postexposure prophylaxis involves three steps, including wound care, passive immunization with human rabies immunoglobulin (HRIG) infiltrated around the wound and intramuscularly at a distant site, and active immunization with human diploid cell vaccine at day 0, 3, 7, and 14.

36-year-old man with a several-day history of flu-like illness, nausea, vomiting, and shortness of breath presents to an ED in New Mexico. He states that his symptoms began approximately one week after cleaning out his barn where he noticed a large amount of mouse droppings. The most common cause of death from this illness is due to failure of which organ system? Cardiac Hepatic Neurologic Pulmonary Renal

Correct Answer ( D ) Explanation: This patient is suffering from hantavirus syndrome, a viral zoonosis caused in North America by the Sin Nombre virus. It is transmitted by a number of rodent types, with the deer mouse being the most common vector in the Southwest United States. It is transmitted by inhalation of dried rodent excrement or, less commonly, by a rodent bite. It begins with a prodromal phase lasting three to four days with vague flu-like symptoms (typically including dyspnea without cough) and rapidly develops into hantavirus pulmonary syndrome, characterized by pulmonary edema, hypoxia, and hypotension. The mortality rate for this illness is 50%-70%.

23-year-old man who has unprotected, receptive anal intercourse presents to the ED with two weeks of worsening rectal pain and dyschezia. On exam, he has numerous ulcers in the anorectal area and a crop of grouped vesicles containing clear fluid on an erythematous base. The surrounding skin shows no sign of cellulitis or abscess. Which of the following is the most appropriate next step? Refer the patient to a surgeon for operative intervention Send a serology test Send a Tzanck smear Treat with acyclovir

Correct Answer ( D ) Explanation: This patient is suffering from herpes simplex proctitis, a sexually transmitted infection. Diagnosis is clinical. Patients present with a painful vesicular rash on an erythematous base. Constitutional symptoms and lymphadenopathy are common. Treatment with antivirals shortens the duration of illness, decreases viral shedding, and improves constitutional symptoms. Commonly used antivirals include acyclovir, valacyclovir, and famciclovir. Referral to a surgeon (A) may be needed in cases of Condylomata acuminata, but for herpes proctitis, primary management involves antiviral therapy. Serology (B) can also be sent, but a significant proportion of the adult population will be seropositive with no history of disease, making results less useful in confirming the diagnosis. If the diagnosis is unclear, a Tzanck smear (C) can be sent after puncturing a vesicle and collecting fluid. However, a negative result does not rule out herpes.

6-year-old immigrant boy from Bangladesh presents with fever and rash. The fever started 3 days ago followed by a rash which started on the head and spread to the rest of the body. Examination reveals a well appearing child with a maculopapular rash and posterior cervical lymphadenopathy. Which of the following is the most likely causative organism for this disease? Measles virus Mumps virus Parvovirus B19 Rubella virus

Correct Answer ( D ) Explanation: This patient presents with classic symptoms of Rubella or German measles. Rubella is a mild, febrile illness that is associated with a diffuse maculopapular rash. The patient may also present with headache, malaise and posterior cervical and postauricular lymphadenopathy. Rubella is transmitted through respiratory secretions. It is highly communicable. The rash generally begins on the head and face and progresses downwards ("showering" rash). The disease itself is typically self-limited, however, the consequences of maternal infection during pregnancy can be severe for the fetus. Mumps (B) is characterized by infectious parotitis. Parvovirus B-19 (C) causes erythema infectiosum, which typically does not have a fever. Measles (A) is characterized by fever cough, coryza and conjunctivitis followed by rash and Koplik's spots.

76-year-old patient presents to the emergency department in January with 3 days for fever, myalgias, cough and dyspnea. On examination, she has a temperature of 38.6ºC, heart rate 110 beats per minute and blood pressure of 110/62 mm Hg. Which of the following laboratory studies can best help differentiate between a bacterial and viral source of infection? AAbsolute neutrophil count BC-reactive protein CErythrocyte sedimentation rate DProcalcitonin

Correct Answer ( D ) Explanation: This patient presents with signs of sepsis (i.e., 2/4 SIRS criteria with a presumed respiratory source of infection). Differentiating between a bacterial and a viral source of infection can help guide treatment and avoid overuse of antibiotics which can lead to bacterial resistance and significant patient morbidity. Procalcitonin, a precursor of calcitonin, is released by the parenchymal cells in response to bacterial infections. It is not found in healthy patients and is down-regulated in patients with viral disease. In recent trials, using procalcitonin levels to guide antibiotic use in patients with respiratory illnesses has resulted in a decrease in antibiotic exposure without an increase in mortality or treatment failure. Some studies have also shown that procalcitonin levels correlate with disease severity. A procalcitonin level of greater than 2 ng/mL is generally used as a marker of significant bacterial infection.

26-year-old woman with a known history of AIDS presents to the ED for strange behavior, according to her boyfriend. Reportedly, she complained of a headache for a few days prior and then began acting bizarrely. In the ED she has a temperature of 38.5°C. Neurological examination is remarkable for word-finding difficulties accompanied by episodes of clanging and echolalia, along with decreased attention span, recall, and consolidation. A contrast CT scan of the brain reveals multiple ring-enhancing lesions without evidence of midline shift. Which of the following is the most appropriate next step in management? Consult neurosurgery for a brain biopsy Obtain an MRI Treat with dexamethasone Treat with pyrimethamine and sulfadiazine Treat with trimethoprim-sulfamethoxazole

Correct Answer ( D ) Explanation: This patient with AIDS and altered mental status most likely has cerebral toxoplasmosis, the most common cause of focal encephalitis in patients with AIDS. It is often accompanied by fever, headache, altered mentation, focal neurologic deficits, and seizures. It is caused by the protozoa Toxoplasma gondii. The initial diagnosis is based on history, physical, and head CT scan. The appearance of multiple ring-enhancing lesions on contrast-enhanced head CT scan is pathognomonic. Treatment should be initiated with pyrimethamine and sulfadiazine. Some regimens also include folinic acid.

Which of the following patients is at greatest risk of developing West Nile meningoencephalitis? 22-year-old man status postrenal transplant 3-year-old girl who is unvaccinated 58-year-old man with diabetes and hypertension 82-year-old woman with dementia

Correct Answer ( D ) Explanation: West Nile virus is a zoonotic infection that first appeared in the U.S. along the eastern seaboard in 1999 but can now be found nationwide. West Nile encephalitis (WNE) is endemic in the Middle East, Africa, and Asia. Birds serve as the primary host, and it is transmitted by the bite of a mosquito. WNE usually occurs in the summer, when mosquitoes, wild migratory birds, and humans are in close proximity outdoors. Most people infected with West Nile virus are asymptomatic. When present, symptoms are typically mild and include fever, headache, and fatigue. Severe disease, however, can cause central nervous system manifestations including meningitis, encephalitis, and myelitis. The biggest risk factor, by far, for neuroinvasive West Nile disease is advanced age.

A 53-year-old man presents to the ED with a two-day history of fever, diffuse joint pain, and mild headache. He notes a rash that began on his wrists and ankles and has spread to his torso. He lives in a rural area in Arkansas and often goes hunting. Vital signs are temperature 39.7°C, blood pressure 125/80 mm Hg, heart rate 117, respiratory rate 16, and oxygen saturation 99% on room air. His skin exam shows a diffuse maculopapular rash on his torso and abdomen, with a petechial rash on his distal extremities, including his palms and soles, as seen in the image above. Which of the following organisms is most likely the cause of this patient's diagnosis? Anaplasma phagocytophilum Borrelia burgdorfei Coxiella burnetii Ehrlichia chaffeensis Rickettsia rickettsii

Correct Answer ( E ) Explanation: This patient has Rocky Mountain spotted fever (RMSF). The infecting organism is Rickettsia rickettsii, which is spread by the female Dermacentor tick. Children ages five to nine years have the highest incidence of infection. The disease was first identified in Idaho and Montana (near the Rocky Mountains) but is now a misnomer because the highest frequency of cases is seen in the Carolinas, Oklahoma, and Virginia. Clinically, it is characterized by fever, rash, and tick bite, although the presentation can be highly variable. Typically, fever and myalgias followed by a severe headache and gastrointestinal symptoms precede the rash (which usually develops two to six days after the onset of the fever). The rash begins peripherally and then moves to the trunk, (centripetal spread) appearing first as a maculopapular eruption and later becoming petechial. Palms and soles are typically involved.

A 73-year-old woman with an indwelling Foley catheter is sent for evaluation from the nursing home because of fever. Which of the following is a criterion for systemic inflammatory response syndrome (SIRS)? Heart rate 86 Lactic acid 4 mg/dL Respiratory rate 22 Temperature 37.9°CCorrect Answer ( C )

Explanation: Systemic inflammatory response syndrome (SIRS) is the systemic inflammatory response syndrome occurring most commonly in response to an infection. SIRS plus a source of infection is the definition of sepsis. Sepsis is an increasingly common cause of ED visits and the tenth leading cause of death in the US. In response to the infectious insult, the host's body activates a number of inflammatory cascades as part of the host response. As patients become sicker, severe sepsis develops with the presence of organ dysfunction in the setting of an infection. The respiratory rate of 22 above fulfills one of the SIRS criteria.

Question: Which two organ systems are mainly affected by early disseminated Lyme disease?

nswer: Neurologic and cardiac involvement define early disseminated disease.

31-year-old woman presents to the emergency department for vaginal lesions. She has had them for one week and she thinks they are warts. She denies any pain. Physical examination shows flat, white, velvety, wart-like lesions on both sides of the vulva and perineum. You ask if the patient ever had a sexually transmitted disease or other types of vaginal lesions and she indicates "yes, but it went away on its own a few weeks ago". What is the appropriate test to perform to make a diagnosis in this patient? Biopsy a lesion and send for human papillomavirus testing Rapid HIV test Rapid plasma reagin test Tzanck test

orrect Answer ( C ) Explanation: The wart-like lesions presented in this question are consistent with condyloma lata, lesions that are found in secondary syphilis. Syphilis presents in three phases: primary, secondary, and tertiary or latent syphilis. The disease may be diagnosed in any of the stages. Primary syphilis is characterized by a painless chancre with indurated borders on the penis or vulva. The lesion resolves spontaneously. The lesion that this patient reports previously was likely due to primary syphilis. Secondary syphilis develops three to six weeks after the primary stage and is characterized by rash, nonspecific symptoms such as fever, sore throat and headache, and genital warts called condyloma lata. Condyloma lata are large, raised, flat, grey or white lesions. They appear in warm, moist areas such as the mouth and the perineum as a manifestation of the secondary stage of syphilis. The lesions can look very similar to condyloma acuminata but are usually more white and flat. Tertiary syphilis develops in about one third of patients after secondary syphilis and may occur three to twenty years after the initial infection. Manifestations of tertiary syphilis include meningitis, dementia, neuropathy and thoracic aneurysm. The causative agent of syphilis is the spirochete Treponema pallidum. Nontreponemal tests include the Venereal Disease Research Laboratory (VDRL) and the rapid plasmin reagin (RPR) tests. These tests detect nonspecific antibodies to cardiolipins, which are released as a result of infection. The VDRL and RPR are used as screening tests. Tests may not become positive until approximately one to four weeks after the initial lesion, the chancre, appears.


Set pelajaran terkait

Environmental Science: The Earth's Atmosphere

View Set

Physical Geography of the US and Canada

View Set

BL2 - CH 40: Corporate Directors, Officers, and Shareholders

View Set

what are the advantages and disadvantages of raising pets ?

View Set

Chapter 24: Diuretic therapy and Drugs for Kidney Failure

View Set

Understanding Relationships in the 21st Century Exam 1

View Set

musculoskeletal & cast care study cards exam 4

View Set

Module 13 TEST - Wellness - Regular Psychology - PRE-test

View Set